Sunteți pe pagina 1din 82

DEVELOPMENT OF LIABILITY BASED ON FAULT Case: Anonymous (1466) No need to prove intent - P is entitled to recover (regardless of intent or circumstances)

Case: Weaver v. Ward (1616) Question of negligence enters Gradual separation between negligence and intent Soldier shot another accidentally, and held responsible b/c liable for injuries directly caused by your actions, unless utterly without fault. **added possible excuse for D -if someone else caused the action with his object. Case: Brown v. Kendall (Mass, 1850) - Battery Claim Trespass --> assault &battery - while hitting two dogs to stop them from fighting d struck p - verdict for p No intent to harm - use of ordinary care - If D actions were unintentional and he acted with due care, not liable. Makes act involuntary and unavoidable w/in the circumstance The burden of proof is on the P to show that the D didnt act w/ necessary care or that he did with intent. - P failed to sustain the burden of proof Extraordinary care to be used for acts that were unnecessary Blessed be the risk takers, for they will not inherit the loss, unless they are strictly liable, intentional or negligent Case: Cohen v. Petty (1933) - Negligence Claim P riding as passenger in d's car when d fell ill and lost control of the car - p failed to show any actionable negligence, Pg 12 n.1-3 When injury results from something that is unforeseeable there is no liability for negligence. Case: Spano v. Perini (1969) - Strict Liability Claim - liability without 'fault'(?) P property damaged due to blasting on nearby property of d's -p relied on absolute liability due to action of using blasting materials Strict liability applies (no need to show intent or negligence) because of abnormally dangerous activity of blasting - Mirrors Anonymous case - no need to prove intent -the question is who should pay not whether the act is actually unlawful 1. INTENTIONAL TORTS -Seven intentional torts -Look at the supersensitive p - are we taking super sensitivities into account - no treat the p as an average person - one exception is where the d knows of the sensitivities (seldom happens) Prima Facie case for Intentional Torts, P must prove: 1. Act by Defendant-the act required is a volitional movement by D. 2. Intent- Must intend the act that amounts to or produces the unlawful invasion and the intrusion must be the virtually inevitable consequence of the willful act. a. Specific intent-the goal in acting is to bring about specific consequences b. General intent-the actor knows w/ substantial certainty that these consequences will
1

result c. Transferred Intent: -intent can be transferred from person to person -intent can be transferred from tort to tort (within the intentional torts) a. Only applies where the intended tort and the actual tort is assault, battery, false imprisonment, trespass to land, or trespass to chattel b. Talmadge v Smith-D threw stick at 1 boy and hit another. Liable. c. Rule: transferred intent cannot be claimed if the original intent is fulfilled d. It is the intent to bring about the consequences of the tort and not injury. Thus, a person may be liable for unintended injuries from the consequences of the tort. a. Vosburg v Putney-schoolboy kicked another lightly boy didnt feel it. Prior condition lit up and causes $2500 injuries. e. Motive impels a person to achieve a result. Intent denotes the purpose to use a particular means to cause that result. Therefore, even if a person acts w/o a hostile motive or desire to harm, or even when he is seeking to aid the P, he may be liable. a. Mistake does not absolve a person from being held liable for intentional tort. i. Ranson v. Kitner-killed dog mistaken for a wolf and held liable f. Everyone is capable of intent in intentional torts-even minors and incompetents. 3. Causation-the Ds conduct must have been a substantial factor in bringing about the injury. *the courts are more willing to extend the chain of causation here than by negligence.* a. Garratt v. Dailey-child pulled chair out from old woman and held liable. 1-BATTERY Restatement 2d of Torts (1965) Pg 33 D intends harmful/offensive contact to the P & harmful contact results either directly or indirectly - remember to take into account the context of the act (Wallace v. Rosen) a. Battery Prima Facie case, need to prove: 1) harmful/offensive contact by the D i. Judged by whether it would be harmful or offensive to the reasonable person of ordinary sensibilities. ii. A contact is deemed offensive if the P hasnt consented to it. 2) to the Ps person i. anything connected to the Ps person is viewed as part of the Ps person ii. Fischer v Carousel-D snatched plate from P and held liable 3) Intent by the D to bring about the contact to the Ps person or Substantial Certainty - an occurrence is obviously 'intentional' if the actor desires to bring it about - but tort law also calls it intentional if the actor didn't desire it but knew with substantial certainty that it would occur as a result of his action. The consequences do not have to be intended or substantially certain or even foreseeable. 4) Causation i. Contact can be direct or indirect (if he sets in motion a force that causes contact to the P Garret v. Dailey) b. Apprehension is not necessary-a person may recover for battery even though he is not conscious of the contact when it occurs c. Actual Damages not required-p doesnt need to prove actual damages to establish a prima facie case. P can recover at least nominal damages even if he suffered no severe actual damages. If malice, punitive damages would be awarded.
2

a. an intentional invasion of personal space constitutes a battery-doesnt need to be physical injury Indirect Battery Case: Garrett v. Dailey (WA SC, 1955) Facts: p claims d pulled chair out from under her sister, causing her sister to fall and sustain injuries D claimed it was a misunderstanding - did not intend for p's sister to injure herself when he moved the chair, p's sister not testifying Restatement clause (a) sec 13 - Character of Actor's Intentions - Distinction between a volitional act but not a malicious act - act must be done with the purpose of causing the contact or apprehension or with the knowledge on the part of the actor that such contact or apprehension is substantially certain to be produced. No intent to harm necessary Case: Vosburg v. Putney (1891) D a schoolboy kicks P - jury finds that although d intended to kick p he did not intend to harm him. Nonetheless p suffers severe injuries - D's intent or lack of intent to harm is irrelevant as long as D intended to kick P Reasonable Knowledge of the risk - to separate negligence or intent Case: Spivey v. Battaglia(SC FL, 1972) Friendly unsolicited hug given with such force that it caused serious injury - P sued D for negligence and assault and battery D claimed only assault and battery - since statue of limitations had run out on this - and asked for summary judgment Court found that there was 'no substantial certainty that the hug would cause the injuries' and found for negligence - st of lim had not expired on this cause of action (intent is defined as reasonable knowledge of the risk ) foreseeability and volition Pg 23 n.5 Foreseeability important in intentional torts? - no, looking at substantial certainty of act and its tortious consequences Question: What did the actor know and when did she know it? And Intention and negligence distinguished - what difference does it make? Minimum Amount of D's Intent or Knowledge of Outcome to be Proven by P Case: Ranson v. Kitner (Ap Ct IL, 1889) dog mistaken for a wolf, and killed the dog-tort of trespass to chattel. D held liable This was really a mistake, and we ordinarily differentiate mistakes and intent; but sometimes mistake seems to be that it really may be intentional. Issue: is a good faith mistake nonetheless intentional? Our cases (since Brown v Kendall) require the universe of potential P to be more careful like the older rule required D to be more careful. Because until a P can show that intent/fault, hes left with the loss. In cases, think whether the D crossed the line between liability and non-liability (not the Ps actions right now) Holding: Mistake is not the opposite of intent! Even honest, lawful mistakes can bring a D into the line of liability.
3

This was not risk creation - which implies negligence - this is substantial certainty that the act of shooting at animal would kill that animal - mistake of what the animal was doesn't matter P has to prove d's purpose to do something that then turned out to be tortious It was enough for the D to have purpose, knowledge, or substantial certainty that a result will occur even if the outcome is unexpected (result has to be a tortious invasion of the p's protective interests) even for good faith mistakes Insane Person can be held Liable for Intentional Tort Case: McGuire v. Almy (SC Mass, 1937) P taking care of D when D had a violent attack of insanity and struck P with a chair while P was trying to secure her, judgment for p Intent is still there even though person may be insane Compare to Weaver Why are people who are not criminal still held liable - if able to form intent then liable for tortious results of intended actions Public Policy Reasons: 1-actor should bear the burden of paying for damages 2-guardians are encouraged to take more care of watching over insane person Transferred Intent Case: Talmadge v. Smith Smith attempting to get 2 boys off of his shed threw a stick at one boy but hit a third unseen boy-P. P sued. Judgment for P - even if D intended to commit tort of assault he ended up committing tort of battery Use of Physical Force Case: Cole v. Turner (Nisi Prius, 1704) Defines Battery as: 1-least touching of another in anger is battery 2-2 people meet in a narrow way and touch without design of harm or violence is not battery 3-if there is violence by one to another there is then battery -intent to do bodily harmDefense - Justification for Touching Case: Wallace v. Rosen (Ind, 2002) P was at school during unexpected fire drill and claims D pushed her down the stairs - D claims she only touched P's back while telling her to get moving because of the fire -battery defined as an intent to invade another's interests in a way that the law forbids Changed slightly to take into account a "crowded world' - reemphasizes intent to do bodily harm Battery is an exclusively intentional tort Liablity for battery based on substantial certainty or policy reasons Wallace uses the black letter law: the least touching of another in anger is battery Must rely on circumstantial evidence to discern if intent was expression of anger -absent expression of anger then consent is assumed to all ordinary contacts which are customary and reasonably necessary in life - here the touch was justified Offensive Contact to P's Person
4

Case: Fischer v. Carrousel Hotel (Tex, 1967) P suing for assault and battery - hotel d's employee snatched plate from his hand and made discriminatory statements loudly - Jury found for p "personal indignity" harmed: Snatching the plate from p - d doesn't need to touch p's person directly - but also anything part of the person at the time The essence of battery is touching - so in this case there is still touching The addition of the racial epithet increased the battery charge - if there had only been a racial epithet but no contact then would not be battery 2-TRESPASS TO LAND Prima facie case: a) An act of physical invasion of Ps real property by the D a. Protecting the interest of exclusive possession of realty, so just need a physical invasion i. D doesnt need to enter on the land-could throw rocks, push people on ii. Lawful right of entry expires b. If no physical object enters the land, its nuisance or strict liability b) Intent by the D to bring about a physical invasion a. Dont need intent to trespass-intent to enter the land is sufficient c) Causation-need to show that Ds act legally caused the physical invasion. Land is the surface, air space, and subterranean to the depth and height that the P makes beneficial use of such space. Every unprivileged entry onto the land of another is trespass regardless of the amount of damages. o Dougherty v Stepp- D entered Ps land to survey it but made no marks. Unauthorized entrance constitutes trespass, and P awarded damages, no matter how minimal they may be. anyone in actual or constructive possession of land may bring an action for trespass. Damages do Not have to be Shown for Intentional Trespass to Land Case: Dougherty v. Stepp (NC, 1835) D unknowingly trespassed onto enclosed land of P and started surveying it but left no evidence. Verdict for D - although intent found, there was no interference with or damage to land The law infers some damage - only has to be an "unauthorized entry into the close of another." Mistake Not an Excuse Case: Serota v. M & M Utilities - Trespass to Land by oil distributor Pg 24 n.2 Mistake-thought it was empty - but not an excuse from liability Still a trespass because although he didn't know the property would flood he can still be culpable of an intentional tort since he was substantially certain that his act could result in the invasion of a protected interest - even if it was a reasonable mistake. 3-TRESPASS TO CHATTELS Prima Facie Case: a) An act of the D interferes with Ps right of possession in the chattel a. Protects a persons right to possess his chattels, so any interference will suffice. Usually 2 forms: i. Intermeddling-directly damaging Ps chattels
5

ii. Dispossession-depriving P of his lawful right to possession of his chattel b) Intent to perform the act bringing about the interference a. Intent to do the act is sufficient-dont need to prove intent to trespass b. Mistake as to the lawfulness of Ds actions is no defense (mistaken belief that D owned the chattel) c) Causation a. The interference with the chattel must have been caused by the Ds act or set into motion by the D d) Damages Chattel owner must prove more than nominal damages-there needs to be actual damages By dispossession, the loss of the chattel is deemed to be an actual harm. Permanent harm or temporal loss Case: Glidden v. Szybiak (NH, 1949) P-Child bit by a dog on D's store porch and sues. D said that the child was a trespasser and was also committing trespass to chattel (chattel-property)since she was pulling the dogs ears. Restatement Pg 76 defines Trespass to Chattel The lower court found that a child cannot be guilty of contributory negligence, and, therefore, cannot be guilty of trespass either. -Seems wrong to let a child off just b/c his age. Here let off b/c didnt injure the dog (temporal loss or permanent harm) 4-ASSAULT Prima Facie case: 1) An act by the D created a reasonable apprehension of an imminent touching to the Ps person 2) Intent by the D to cause the apprehension to the P: 1- intent to cause apprehension 2-intent to cause the contact itself 3) Causation-apprehension needs to have been legally caused or set into motion by the Ds act either directly or indirectly. Assault not suffered unless the victim knows of their dignitary invasion -No hostility is required as with pranks - Apprehension doesn't equal fear -Transferred intent -any words should be accompanied by some overt act but sometimes an overt act not necessary to confirm creation of apprehension -words can also negate an intent to commit assault -Threat has to be imminent and not a future threat of assault - d has to demonstrate present ability to harm immediately -unlike with a battery where p doesn't have to be aware, w/ assault p must be aware - has to believe that harm will happen (doesn't have to be a fact that he could be harmed) -Has to be a reasonable apprehension, but if d intends to create apprehension in a timid person this can still be viewed as assault -must be a threat to p and not to a third party Conditional Threats -unless legal right to threaten p conditionally then can be seen as an assault -if legal right to threaten then threat of unreasonable force is taken into account
6

Privilege of making p have to choose between contact and compliance Assault is not an attempted battery Even abandoned battery attempts will stand as assault if the d's actions created apprehension Words alone are not sufficient to constitute an assault - words may have to be accompanied by some Words may Negate Assault Case: Tuberville v. Savage (1669) P gets into an argument with d and puts his hand on his sword and says: "if it were not assize-time I would not take such language from you" - His words negate overt act of pulling sword since threat of harm is not imminent anymore Damages not required - don't need to prove to make out prima face case - p may recover nominal damages , if malice is shown punitive damages may be awarded. Case: I de S et ux v. W de S (1348) D went to establishment of p's beat at their door with a hatchet, and threatened M with hatchet raised when she asked him to leave. at first , 'no trespass since no harm done' But court found there was harm done and damages should be awarded for assault. Public policy - prevents others from attempting such threatening acts, or creating apprehension of harmful or offensive contact Imminent Threat Requirement Case: Western Union v. Hill (Al, 1933) p's wife claimed assault by d's employee sapp (respondeat superior) Who attempted to pull her behind the counter of his office and "pet" her, he also stated he would "fix her clock" jury found for p Reversed - not western union's liability - p has to sue against sapp directly Also imminent character of the threat was missing - d could not reach p across the counter 5-FALSE IMPRISONMENT Prima Facie Case: a. An act or omission on the part of D that confines or restrains the P b. To a bounded area c. Intent d. causation Physical Barriers Case: Big Town Nursing Home v. Newman (Tex, 1970) d-nursing home, p-newman P claimed imprisoned for 2 mos at nursing home Pg 41 - direct restraint by one person of the physical liberty of another without adequate legal justification 1-refusal to admit is not false imprisonment 2-no constraints on location 3-but if there is a means of exit then not false imprisonment 4-Exit has to be without peril or danger - or unreasonable 5-if there would be exposure or material harm or if means of escape are unknown to the p
7

6-if p attempts harmful escape and could have remained imprisoned without harm then p not allowed to recover for injuries sustained 7-exclusively an intentional tort - do not have to prove damages -also consider negligence resulting in confinement (unknowingly confine someone) - this would require damages -transferred intent of false imprisonment - if intend to imprison someone and also unknowingly imprison someone else the intent is transferred -if legal justification for confinement (p is committed), even if accidental or mistaken , then not false imprisonment P Required to Be Aware of Confinement Case: Parvi v. City of Kingston (NY 1977) p-parvi d-city P while intoxicated was bought by the police to golf course so that he could cool off p ended up wandering onto a nearby highway and getting hit by a car and injured P had no recollection of the night - Question of consciousness of confinement Victim has to be aware of the invasion on his personal dignity but p's inability to recall later does not mean he was not aware at the time of the confinement - should have proceeded to jury trial to determine p's awareness at time of confinement Dissent - p failed to establish prima facie case - no factual allegations of false imprisonment since he has no recollection P must be aware of confinement but no necessity to recollect or must suffer some harm from confinement P's Desire to Clear Self of Suspicion negates False Imprisonment Case: Hardy v. Labelle's Distr Co (Mont, 1983) P claimed false imprisonment by employer d - accused of stealing Brought to showroom manager's office under false pretenses, Agreed to a lie detector test Court held there was enough evidence to show that p was not falsely imprisoned: -p did not ask to leave the office - stayed to clarify the situation -p was not told she couldn't leave -no threats were made to p to compel her to stay Thus, she was not detained against her will -False imprisonment like assault has to be imminent - threats of future harm will not be enough -Shopkeeper's privilege to detain a suspected thief if detention is brief and suspicion is reasonable Defense of Valid Arrest Case: Enright v. Groves (Col, 1977) p-enright, d-city and officer groves D while on duty saw an unleashed dog heading towards p's house - Unleashed dog is an ordinance violation - D asked p for her driver's license - p refused D threatened to jail her for not producing license, grabbed her arm and said "Let's Go!" P paid bail and later was convicted of ordinance violation Since in this case d made the non-production of the driver's license the cause of taking p to jail, and not the ordinance violation of which p was later convicted for, there was no legal right (privilege) for him to do so - the demand was unlawful and the subsequent arrest was unlawful - false arrest
8

False Arrest - when one is taken into the custody by a person who claims but does not have the proper legal authority, if detainer has a valid warrant or probable cause the it is not false arrest. Conviction of a crime for which one was arrested -can be used as a complete defense to a false arrest claim regardless of unreasonableness of arrest if p actually committed the crime then not false imprisonment Duty to Aid in Escape or Release Case: Whittaker v. Sanford (Me, 1912) P married to religious sect leader, d sect leader P upon leaving sect accepts d's offer to sail on his yacht to America on condition that he will not detain her on the boat, d-refused to secure a way for p to get off the boat upon arrival. P finally able to leave boat with police help -p was physically restrained and not just morally influenced to stay - judgment for p for false imprisonment -intent - if there is substantial certainty that confinement will result then that can suffice as intent Means by which Confinement Enforced: -physically confined -use of explicit or implicit threats - but if p's confinement based on a desire to clear self of suspicion then not false imprisonment -if commands are strictly verbal may not show false imprisonment -threats to harm third party - to harm property - but still must be imminent harm - future harms are not sufficient -assertion of legal authority to confine - if p reasonably believes then can be false imprisonment - it is irrelevant whether the asserted legal authority is in fact valid or invalid - it depends on p's belief -If it is a valid legal authority then has the right to make arrest if clearly stated at the time of arrest -actual submission is necessary Instigation in Arrest Private citizen who participates in false arrest can be held liable -if private citizen files a complaint with police then not held liable for false imprisonment that might result but may be liable for malicious persecution -if d takes an active role and the legal formalities are upheld then not held liable Instigator liability: 1-unlawful arrest occurred 2-d actively aided the arrest (persuaded) Duty to Aid in Escape or Release: P might initially consent to the confinement but d must help p escape or release p if there is duty involved or will be considered false imprisonment - Whittaker v. Sanford

Detainment for Recapture of Chattels Case: Bonkowski v. Arlan's Department Store (MI COA, 1968) D's agent detained P on suspicion of larceny. P sued claiming false arrest and emotional distress. Issue: Did D's agent reasonably suspect the P had stolen items from Arlin's to be able to have the defense of privilege for detaining P?
9

Holding: Unclear - remanded back to Tr Ct for jury to decide reasonableness of D's actions. If D's actions are found reasonable by jury then the detention of P was privileged under the circumstances. Reason: Restatement of Torts gives a defense for false arrest that is reasonable and a short amount of time in order to give shopkeepers the ability to defend their stores and goods. Bonkowski is a customer and events took place outside of store -Unlike Hardy where P was an employee and events took place inside of store - but employee did not make clear that she did not want to be detained. Did Bonkowski resist? No - she was not guilty of shoplifting Could d detain p outside of store? Yes a store keeper can detain a person who he reasonably believes has taken his chattel. Fresh Pursuit of alleged shoplifter occurred right away - limited by reasonableness of the suspicion 6-INTENTIONAL INFLICTION OF EMOTIONAL DISTRESS A complaint for mental suffering caused by outrageous conduct will be sustained if there was no privilege to act in such a manner Prima Facie Case: 1. an act by the D amounting to extreme and outrageous conduct a. To limit abuse of this tort, the conduct must transcend all bounds of decency tolerated by society. b. Conduct that is not normally outrageous will be if it continuous in nature, directed a certain kind of P (kids, elderly, pregnant), committed by a certain type of D (common carrier, innkeeper) c. Ex of outrageous conduct: i. Extreme business conduct-collection ii. Misuse of authority-school bullying pupils iii. Offensive language if there is a special relationship-common carrier and innkeeper iv. Known sensitivity-if D knows that P more susceptible and does, D liable. Ex: sensitive adult, kids, pregnant, old 2. intent or recklessness a. liable for intentional and reckless (disregard for the high probability that emotional distress will result) b. Intention-dont need to show purpose or motive, but you do have to show that the D knew w/ substantial certainty that there would be an invasion to the person of the other 3. causation a. D conduct proximately caused the Ps emotional distress b. Where the D intentionally harms a 3rd person and the P suffers emotional distress b/c of relationship to that person, harder to prove intent and causation. So if cant show prima facie, can prove to establish case: i. P was present when injury occurred ii. P was a close relative of person injured iii. D knew that P was there and was close relative Taylor v Vallelunga-Daughter saw her father beaten. D won b/c didnt know daughter there. c. There is a special liability for the mishandling of corpses of a relative. 4. damages-severe emotional distress a. actual emotional damages must be shown (physical injury not required) b. Punitive damages allowed where D conduct was improperly motivated.
10

State Rubbish Collectors Assn v Siliznoff -Siliznoff collected garbage and was scared into signing to pay OR ELSE! Liable. Found IIMD w/o P showing any physical effects! Harris v Jones-D made fun of Ps stutter. Daily teasing in the workplace. Not held liable. No recovery when the abuse, insult, or profanity is not accompanied w/ a serious threat for physical harm. a. Slocum v Food Fair Stores of Florida-woman verbally abused in store and aggravated a pre-existing heart condition and heart attack. D not held liable b/c mere vulgarities. Extreme and Outrageous Conduct Case: State Rubbish Collectors v. Siliznoff (CA, 1952) p-state rubbish collectors, d-siliznoff P sued d for outstanding payments - D counterclaimed for duress, want of consideration, and assault P states that d's claim of assault cannot hold up because the threats were future threats of harm and not imminent Traynor Court holds - when one without the requisite privilege intentionally subjects another to mental suffering, incident to serious threats on his well-being whether or not a technical assault, if d intentionally subjected p to such distress and bodily harm resulted then d is liable for negligently causing p bodily harm -p's right to be free from negligent interference on his physical well-being -POLICY TORT - restrict anti-social behavior, mental suffering is the damage - required that there be physical injury to stem bogus litigation but the jury can decide if the mental suffering is valid and severe enough Not completely new, adds to the following: Assault cannot apply because the threats were not imminent False imprisonment -no, he went to meeting/ freedom of movement not curtailed Battery - no, no physical toughing Though not a technical assault still have a cause of action due to mental distress caused by another Was physical injury required? - no, although siliznoff claimed physical illness - but court decides not important factor - the severe emotional distress caused is enough to prove damages (pg 65) - Pin down distress at fault laden actions of actor and then can attach pain and suffering. Sometimes even though cases where there were psychic affects still had to prove the physical repercussions - Now take away requirement of physical bodily harm Have to judge the severity of the emotional distress, also judge the outrageousness of the actor's conduct, as well as the actor's intent or recklessness - jury job Offensive Language not Enough Case: Slocum v. Food Fair Stores (FL, 1958) Slocum was verbally abused by d's employee (you stink to me) and claimed emotional distress For severe emotional distress to be proven p needs to show she/he required some sort of medical attention Court decided D does have the right to treat the P the way he did- even though it's rude behavior not illegal behavior (or outrageous, reckless behavior) P actually eventually had a heart attack due to the emotional distress - Sil did not sustain such a great physical effect. More so the behavior of the actor than the effect on the sufferer. Insult v threat: Insult not threat in this case! Sil involved threats. Substantial certainty? - yes
11

Causal link? Not an issue in this case but more so in Harris v Jones (case of teasing) A common carrier - bus company, airline, inn keeper etc. - they were held to higher standard of behavior but not so much today pg 54 n. 5 Restatement 2d sec 46- Emotional Distress 1-conduct must be intentional or reckless 2-conduct must be extreme or outrageous 3-causal connection between wrongful conduct and the emotional distress 4-must be severe distress Actual Severe Distress shown through P Seeking Medical Aid Case: Harris v. Jones (MD, 1977) P stuttered and was teased continuously by hid employer d and other employees Claimed emotional distress resulting in worsening of his nervous condition through the malicious and cruel ridicule of d -personality of individual is taken into account to test outrageousness but court here found proof of distress not sufficient to show severe distress, and the causal connection was also weak.(used restatement) - Couldn't link severity of mental distress with the D's action P claimed his speech impediment and nervous condition worsened due to workplace teasing Pre-existing condition and so the court has to find in this case an insufficient causal link that could not be proved by P The 4 elements are present but have to weigh how much in each case How important is the physical manifestation of the emotional distress? Maybe because it was hard to link distress to D's actions they could not really determine the severity that resulted Intentionality must be proved by P - purposefulness also but more so intention of outcome (outrageousness and extremeness of act) Recklessness also used to calibrate in some jurisdictions Difference between severe threat in Sil and the mere insult and teasing in Harris and Slocum Almost exclusively a VERBAL tort Doesn't matter that response of P is meager IIED to Third Party Case: Taylor v. Vallelunga (CA, 1959) p- daughter alleges she witnessed d beat up her father and as a result suffered severe emotional distress - p presented no physical evidence D was not aware of p's presence when he beat up her father - If d knows a third party may also suffer emotional distress from his intentional tort then may be held liable -no proof the d knew of p's presence so therefore no intent to cause emotional distress - D has to be aware Daughter did not succeed because she could not prove that the actor had substantial certainty that there would be emotional distress of him beating up her dad 2. DEFENSES TO INTENTIONAL TORTS 1-CONSENT If a P has consented to an intentional tort then the D will not be held liable. Lack of consent is part of the P's prima facie case and so he must plead and prove it.
12

Consent does not usually come into play with negligence cases because negligence and strict liability focus on the assumption of risk. Implied Consent a. Ps consent may be implied in a given case/ there are 2 types: i. Apparent Consent-what a reasonable person would infer from the Ps conduct. 1. inferred from usage and custom-ex: daily life-bumped in crowd ii. Consent implied by law-where action is necessary to save a persons life or some other important interest in person or property Ex: unconscious b. Silence and inaction can imply consent to a Ds acts if the circumstances are such that a reasonable person would speak if he objected. i. OBrien v Cunard S.S. Co-People on the ship being vaccinated. P had the shot, Dr said need it anyway and she put out her hand. Directed verdict for D means consent proven. 1. Questioned the consent in this case. Capacity Required: incompetents, drunks, children are incapable of consenting to tortuous conduct. Need consent of a parent or guardian. Criminal Acts-a person cannot consent to a criminal act a. The modern trend is to differentiate b/t acts that are breaches of the peace so the consent is ineffective (street fight) and those that arent so the consent is effective (prostitution) b. Where the act is made criminal to protect a limited class against its own lack of judgment (statutory rape), consent is not a good defense. When one voluntarily engages in illegal conduct which will cause injury, the privilege of consent can be asserted to defend any civil action the P might bring. Maxims: In pari delicto-in equal guilt, the Ds position is better. One should not profit from his own wrongdoing-this maxim also works against the P Allowing people involved in illegal activities to sue brings a benefit to society that it wont allow people engaged in illegal activities to take it too far. Its like an incentive to not participate in illegal activities because they can be sued. If the D goes beyond the consent given and does something substantially different, he is liable. o Mohr v Williams-surgery on the right ear. It was ok, but left had problems so operated on left. Hackbart v Cincinnati Bengals, Inc - An intentional blow during a football game may give rise to liability even though there are rules to the game, Apparent Consent - what a reasonable person would infer from P's conduct Case: O'Brien v. Cunard SS Co (SC MA, 1891) P about to disembark - D's doctor gives her a vaccination even though P says she has already had one. P holds out her hand to show the mark Doctor says he sees no mark and gives her the shot. P sues for battery. D says she consented. D's doctor is not liable for battery because P implicitly consented to his act. P's behavior did not indicate that she was not consenting- she held up her hand and did not vocalize any objection, therefore her actions(or non-actions) implied consent and so D cannot be held liable. Consent can be held to exist regardless of the subjective state of P's mind (versus the objective manifestation of P's consent) - (Objective manifestation can be tested by comparing whether a reasonable person in D' position would believe the P consented) But sometimes P can prove subjective consent Possible battery - harmful and offensive toughing - defense for D was that P consented by not objecting vocally to the shot - but she did hold out her arm
13

"Volenti non fit injuria"(to a willing person no injury is done)- how, where and when do we (or the context, or the "rules") place ourselves outside the protection of the intentional torts? The first chapter of any lawsuit the P must begin the story usually about the d's behavior , but with Cunard we are now in consent land - shifted the focus to the p's behavior Don't need vocalization for consent or non-consent What about the doctor giving p shot regardless of her claim that she had already had one? Med Mal pg 102 n. 6 - informed consent this has shifted from intentional tort to negligent tort What is dif between principle case and the next case (Mohr v Williams) - we are dealing with intentional tort cases but with med mal the concept of negligence comes into place Professional athlete's consent - Consent Implied by Law/Rules Participating in a usually-violent professional sport does not always constitute consent to all injuries from adversaries A player who intentionally attacks or injures his opponent can be held liable for damages CASE: Hackbart v. Cincinnati Bengals(USCOA 10th, 1979) P - Hackbart was hit by player for the Bengals (D) in the back of the head and neck by the player's forearm during a game. D's player admitted to hitting P out of frustration of his team losing. D is liable for injuries sustained by P because D intentionally struck P during the game in violation of the sport's rules. Although some injuries that may be sustained while playing a violent sport are implicitly consented to - there are some acts that are expressly prohibited by that sport (and so not consented to by P), although the sport may or may not pick up on the violation of their rules. When a player intentionally strikes another during a professional game this can be seen as a cause of action regardless of the violent nature of the sport. This does not extend to negligent violation of rules. Even if players consent to rules of the game, any touching outside of the rules can be held liable against the violator Serena Williams - could have been held liable for assault, or intentional infliction of emotional distress Especially with verbal torts things like teasing or insults have to toughen hides a little bit Presence of rules does not imply consent to all kinds of violent touching Exceeding the Scope of Consent If P gives consent to D, D is still not privileged if he goes outside the scope of the consent given. If D invades P's interest in a way that is substantially different from that consented to D will be found liable. The consent to act is different from the consent to consequences of an act. We are focused on the consent to the act regardless of the consequences. For surgery scope-of-consent is very important - consent to one type of surgery usually does not extend to any other type of surgery. CASE: Mohr v. Williams (SC of MN, 1905) P receiving surgery while under anesthesia by D. Prior to surgery P consented to have surgery done on her right ear. During the surgery D found problems with left ear and not the right ear and performed successful surgery on the left ear without informing P, who was still under anesthesia at the time. D can be held liable for technical battery by performing surgery on P that P did not specifically consent to having done. P's consent to surgery on her right ear does not extend to surgery on her left ear and D should have obtained such consent beforehand (although he performed surgery well and in good faith.) Extend consent if there is a serious imminent emergency otherwise the doctor has to obtain consent Patient deemed as a matter of law to have consented if:
14

1-Patient was unable to give consent - unconscious/anesthesia 2-action necessary to save or safeguard patient's life 3-defendant had no knowledge that patient would refuse to consent if conscious 4-a reasonable person would have consented in the circumstances Mistaken Consent Known or Induced by Defendant CASE: DeMay v. Roberts (SC of MI, 1881) P having her baby at her home. D1 was her doctor and went to her house to deliver baby and brought along with him D2 who was not a doctor, a fact which P was not told at time. D2 helped P with delivery. D1 and D2 are liable for battery if P gave her consent to D2 to touch her and D2 and D1 withheld information on who D2 actually was. Although P gave her consent for D2 to be there, she was misled as to who D2 actually was. P would not have consented toD2's present but for the fact that she was mistaken about his identity and D1 nor D2 informed her of her mistake. If D knows P is mistaken or induces that mistake the mistake vitiates the consent. Why was there no consent? - Consent given under false pretenses The fact that the husband of the p seemed to have consented doesn't matter - the p who was touched has an individual tort - doesn't matter if touch not done maliciously Fact that everyone around her approved doesn't mean she did - fraudulent inducement of the consent negates the consent Pg 101 n 3 A women consents to sex with husband without knowledge of his having an affair Sues for battery - her consent obtained by fraud - If there is consent - no liability If liability and consent is the issue then there was no liability Hart v. Geysal Consent to an illegal activity - can estate of dead P still sue other participant (d) Can one give one's consent to an illegal act? no Don't need a contract for court to discover consent (O'Brien) The d would win - P's consent is valid? Or weakens the P's case against D - could also be a policy decision- warning future offenders not to engage in illegal activity. D may have to pay twice (criminal charges and loss to P) | P should not profit from his own wrongdoing Compare to Katko where P did get awarded damages even if he was in the act of wrong doing In pari delicto potior est conditio defendentis - in equal guilt, the position of the defendant is the stronger. Not going to shift the loss. Exceptions of where consent negated : Statutory rape - cause of action sustained Competency taken into account to assess whether consent is given. Consent and other privileges relate to any of the Intentional Torts. Fraud involves looking at what d did to get consent. 2-DEFENSE OF SELF AND DEFENSE/RECOVERY OF PROPERTY a.Defense of Self when a person has reasonable ground to believe he is being or about to be attacked, he may use necessary force as protection against injury. When is the defense available?
15

1. Reasonable Belief-the privilege exists as long as the D reasonably believes that force is necessary to protect himself. Apparent necessity, not actual necessity! a. A reasonable mistake as to the existence of a danger is allowed. 2. Retaliation-when there is no longer a threat of battery, the privilege terminates and the original victim becomes liable for battery if he uses force a. The initial aggressor is not privileged to defend himself against the other partys reasonable use of force. If the party uses deadly force when aggressor didnt, the aggressor may defend himself with deadly force. 3. Provocation-insult, threats, and opprobrious language do not justify the exercise of self defense. a. If the words are accompanied by threats of violence or by an overt hostile act, then oral abuse may amount to a challenge to fight and then would be allowed 4. Amount of Force-can use the amount of force that is or reasonably appears to be necessary for protection against the threatened battery. a. Deadly weapons can only be used if the D has a reasonable apprehension of loss of life or great bodily injury. 5. Retreat-since it is hard to retreat from an assailant, the majority holds that a person may stand his ground and use the force necessary to protect himself. 6. Injury to 3rd Party-if s/o accidentally hurts a 3rd party while defending himself, the privilege of self defense is carried over, like transferred intent, and he will not be liable to the 3rd party. Rape - alone this will constitute a threat of serious bodily injury - use of deadly force allowed b.Defense of Others a.Actor needs to have the reasonable belief that the person being aided would have the right to self defense. i. The D may use reasonable force even if his intervention was unnecessary as long as it was a reasonable mistake b. The defender may use the amount of force that he could have used had the injury been threatened at him. c.Defense of Property Privilege to Defend Property can be exerted with reasonable force and has to be initiated with verbal demands if there is no immediate harm. One may use reasonable force to prevent the commission of a tort against real or personal property. A request to desist or leave must first be made unless it clearly would be futile or dangerous. The defense does not apply once the tort had been committed; however, one may use force in hot pursuit of another who has tortiously dispossessed the owner of her chattels because the tort is viewed as still being committed. Whenever an actor has a privilege to enter the land of another because of necessity, recapture of chattel, etc, that privilege supersedes the privilege of the land possessor to defend her property. Mistake: a reasonable mistake is allowed as to whether an intrusion has occurred or whether a request to desist is required. A mistake is not allowed as to whether the entrant has a superseding privilege, unless the entrant leads the D to reasonable believe it is not privileged. Use of Mechanical Devices: Landowner is privileged to use mechanical devices to protect their property if he would be privileged to use a similar degree of force if he was present and acting himself. The Spring Gun case:
16

CASE: Katko v. Briney (SC of IO, 1971) D owns a boarded up farm house that is unoccupied. House has been broken into numerous times so D sets up a shot gun connected to bedroom doorway that will discharge if someone opens the door (a spring gun). P breaks into the house with another man and sets off shotgun which fires and injures him(blew off his right leg). D is liable for injuries sustained by P. Property owner may not use deadly force to defend his property against a trespasser unless trespasser is committing a violent act or endangering human life that warrants such use of force. Property owner must use mechanical force equal to a force he himself would use. Jury Instructions: includes provision for substantial certainty that the injury would result regardless of the outcome D intended. Dissent: Should be liable only if P can show the D intended to shoot anyone who entered the room. D testified to only trying to frighten intruders not shot them (isn't this still battery though?) What privileges do we have to protect intentional invasions of our property -> (us, our own or our family's persons) Plaintiffs were not only trespassers, they were thieves - but still persons trump property Warning has to be specific (assuming it could mitigate liability) to alert trespassers as to dangers (spring guns or dogs) d.Recovery of Property Basic rule like reentry on land: when anothers possession began lawfully (a conditional sale), can only use peaceful means to recover the chattel. Reasonable force can be used only when in fresh pursuit of one who has obtained possession wrongfully, by theft. Hodgen v Hubbard- P got stove on credit by misrepresenting his assets. D ran after to forcibly take it back. This shows that the law does respect property because recognizes a privilege to recapture chattels when the owner has been defrauded or his rightful possession and he pursues the wrongful taker of his goods in fresh pursuit. 1) When is defense available? a) Timely demand to return the chattel is first required unless clearly futile or dangerous. b) Can only recapture from a tortfeasor or some 3rd persons who knows or should know that the chattels were tortiously obtained. You cannot use force to recapture chattels in the hands of an innocent party. 2) Entry on land to remove chattel: a) On Wrongdoers land When chattel is on the wrongdoers land, the owner is privileged to enter and reclaim it at a reasonable time and in a reasonable manner, after made demand for return. b) On land of an innocent party When chattel is on the land of an innocent party, the owner may enter and reclaim it at a reasonable time and in a peaceful manner when the landowner has been given notice of the presence of the chattel and refuses to return it. Right to recapture supersedes right to defend property. c) On land through owners fault There is no privilege to enter the land and the chattel may only be recovered through the legal process. 3) Generally no mistake is allowed. Shopkeepers have a privilege to detain for a reasonable period of time individuals whom they reasonably believe stole.
17

Bonkowski v Arlans Department Store- P accused of taking jewelry and suing for false imprisonment because the store detained her 30 ft from the store. (broadens the privilege because the customer left the premises) Possession obtained by Fraud: Must be reasonable force used when recovering property/never deadly force Although while the recovery is taking place use of deadly force in self-defense may happen if there is resistance. If wrongdoer obtains property through fraud such act is wrong from the beginning and owner may attempt to recover property if discovered promptly and may use reasonable force. CASE: Hodgeden v. Hubbard (SC of VT, 1846) P - Hodgeden(?) purchased stove from Don credit. D quickly realized the P had bad /no credit and had committed fraud. D chased after P and because P put up a lot of resistance, and drew a knife, D had to forcibly hold down P to recovered stove. D is not liable for battery. The property was fraudulently acquired by P therefore the D had a right to recover the stove if done with reasonable force and if it would not disturb the peace. In this case D used reasonable force, and the disturbance of peace was caused by P (who in addition to resisting also pulled a knife). Under certain circumstances a p does have the right to take the law into their own hands with a modicum of violence - The d's did start the fight but p escalated it by pulling knife Goes against the peaceful resolution by the courts basis of torts - These privileges are exceptions 3.NECESSITY In certain cases a D is privileged to cause harm to P, usually due to unusual exigencies, even though P is completely blameless. A person may interfere with the real or personal property of another when it is reasonably and apparently necessary to avoid threatened injury from a natural or other force and when the threatened injury is substantially more serious than the invasion that is undertaken to avert it. D allowed to harm P to avoid harm to third party (Public Necessity) or himself (Private Necessity) In the public case usually the D does not have to pay for the damage, for a private necessity D usually does have to pay for damages. 1) Public Necessity-when the act is done for the public good Surocco v. Geary-D blew up P building to prevent a fire from spreading. J/D. o Sacrifice the property of an individual for the greater interest of society 2) Private Necessity-when the act is done solely to benefit any person/property from destruction or serious injury. An actor must pay for the injuries he causes. Vincent v. Lake Erie Transp Co- Ship owner left ship and the P took care of the ship when it otherwise would have been lost in the storm. J/P/A. We have 2 competing interests at stake-2 property interests. The court says that the D made a decision based on the relevant efficiency of the outcome: he wanted to save his more valuable ship at the expense of damaging the dock, so liable. Public Necessity - has to appear reasonably necessary to interfere with land or chattel to prevent a disaster to the community or a substantial number of people. CASE: Surocco v. Geary (SC of CA, 1853)

18

P's house in the vicinity of a fire. P is removing items from house. Fire has already passed over house but D determines that there is still danger of the fire spreading and the only way to prevent that is to blow up P's house and does so. D is not liable for damage to P's property sustained while trying in good faith, and with apparent necessity, to stop the spread of fire because he acted out of necessity. D had to act out of necessity and so in this case was privileged to cause damage to P's property due to the emergency of the situation. Private Necessity CASE: Vincent v. Lake Erie Trans D navigated and docked his ship at P's dock during a storm - because of horrible storm conditions the boat caused damage to the P's dock. P (Vincent) sued D for damage to his dock caused by the D's boat during the storm. Although the necessity of docking during the storm, the D is still liable for the damages caused to P's property. P's property was not going to cause harm to anything else that would warrant damage to it or was not in danger itself So D is liable for damages - a case of private necessity Compared to Private Necessity of Ploof v. Putnam - not a case of trespass because of ps private necessity to protect passengers on board - the ds in this case were held liable for unmooring the boat and letting it drift and become damaged at sea. No negligence and no intentional tort - no fault (technical trespasser - no necessity - no negligence reasonable as a matter of law - his benefit should not be another's loss, so we shift the loss) Cost-benefit analysis - damage to boat more than damage to dock keeps this in a.Anomalous - no fault! b.economically efficient a. If the ship had been worth less than the dock damage ds would not have damaged dock allegiance to your own property kept in check. 4.Arrest - Bonkowski, Hardy v. LaBelle, Parvi, Enright v. Groves 3. NEGLIGENCE HISTORY Negligent at first was generally just used to refer to the breach of any legal obligation First was used against common carriers and doctors - Then became apparent there could be liability for negligent acts separate form intentional acts ELEMENTS for Prima Facie Case: Negligence is used to describe the conduct of the actor - but cause of action requires more: a. Duty - Actor is to use reasonable care to protect others from unreasonable risks b. Breach - failure to conform to standard - there can be negligence but no liability because there was no duty c. Causation- there must be a causal link between the act and the resulting injury. There are two types : Causation in Fact and Legal (Proximate) Causation d. Damage - actual loss or injury to another's interest i. Pleading and proof of action essential to case for negligence Defendant can also claim contributory negligence to call into question p's negligence contributing to result
19

1-DUTY Obligation recognized by the law requiring an actor to conform to a certain standard of conduct for the protection of others against unreasonable risks. 1. ESTABLISHING A DUTY a.Unreasonable Risk of Harm In order to demonstrate that d's conduct failed to meet the duty of care imposed on him the p must show that the d's conduct imposed an unreasonable risk of harm on the p/or class of p's (conduct at the time it occurred not the result) b.Inherently Dangerous Objects Objects that are not inherently dangerous or pose slight danger - leaving them around will not be considered negligent conduct even if it turns out that unexpectedly they cause harm. - risk is to be evaluated as it reasonably appeared before the accident CASE: Lubitz v. Wells (Superior Court of CT, 1955) P - Lubitz is suing D- Wells the father of boy who hit P's daughter with a golf club while playing. P claims D is liable for negligence. D knew the golf club was in the play area and that his son would play with it and could get hurt or hurt others with it - but he did not remove the golf club from their access or warn his child about dangers of swinging the club when others are close by. D is not liable - allegations of P not sufficient to establish negligence on part of D. The danger of a golf club is too obvious to attach negligence. Emphasis on benign nature of golf club but may also be extended to shotgun left in backyard of a more rural area that that is common in What combination of factors determines if the actor's injury-causing behavior might be negligent? Can a child be culpable of a negligent act as opposed to the intentional torts? May be why the father is the D. But also holding Father liable for leaving golf club out. Foreseeability of risk - Is this the same as substantially certain? No, father had no substantial certainty that P would get hit with club, But could have foreseen risk of leaving gulf club out in back yard where children might play with it. Although a benign instrumentality there had to be some foreseeability - could have been a stick, or an axe - not as unexpected but still not as expected Foreseeability might result in a range of risks - this kind of potential may exist even if a benign instrument is left in back yard - even if it is reasonable - what is the standard of care(duty) attached to the act(? ) Lubitz decided by the court as a matter of law - why? No liability why - no duty Unreasonable Risk because of Remoteness of Risk Held not to be negligent conduct because the risk of such a heavy frost was so remote as to not be the kind of risk an ordinary prudent person would foresee /guard against in doing the work Case: Blyth v Birmingham Waterworks Co. (Court of Exchequer, 1856) Pg 131 Defendant, Birmingham Waterworks Co., maintained water pipes and by statute, was required to maintain fire plugs in the pipes. Defendant maintained said fireplugs as required by law. One winter, however, there was extreme frost and one of the fireplugs in the pipe located close to plaintiffs (Blyth) house failed and caused flooding which resulted in damages to plaintiffs property. Plaintiff sued defendant for damages and the jury found the defendant negligent and awarded damages. Defendant appealed. Decision: judgment for the D
20

Issue: Whether defendant was negligent for failing to take precautions against an event that was unforeseeable (i.e., unusual and extreme frost)? The Court held that in order for one to be negligent, he must fail to do what a reasonable person would have done under the circumstances; or he must do what a reasonable person would not have done under the circumstances. Here, the frost was an unforeseeable event that was not within the contemplation of defendant and the resulting damage was a mere accident for which defendant cannot be held liable. Doing so, according to the Court, would be equivalent to making the defendant an insurer for plaintiffs property. Rule: The Court defined negligence as follows: "Negligence is the omission to do something which a reasonable man, guided upon those considerations which ordinarily regulate the conduct of human affairs, would do, or doing something which a prudent and reasonable man would not do. The defendants might have been liable for negligence, if, unintentionally, they omitted to do that which a reasonable person would have done, or did that which a person taking reasonable precautions would not have done. Blyth - pipes unexpectedly froze and caused a leak - D prevailed - jury verdict was overturned Was it unforeseeable that pipes might leak and flood P's property? If No foreseeability can't hold anyone liable for not taking reasonable precautions What is the common thread? Joins with forseeability to create the negligence formula c.Balancing Test : B < L x P B equals the burden which the defendant would have to bear ro avoid risk, L equals the gravity of the injury and P equals the probability that the harm will occur from the d's conduct ***CASE: US v. Carroll Towing Co. (USCOA 2d, 1947) A barge broke away form a tug boat, and it rammed against a tanker and the propeller of the tanker made a hole in the barge, and the barge lost its cargo and sank. Connors is trying to recover the value of his barge form Carrol, and Caroll is trying to reduce the damages because Connors bargee was absent. The trial court reduced Connors damage recovery, because the barge would have remained afloat if the bargee had been aboard to sound a warning. Issue: Whether the absence of a bargee or other attendant will make the owner of the barge liable for injuries to other vessels if she breaks away from her moorings The owners duty to provide against resulting injuries is a function of 3 variables: a. the probability that she will break away (P) b. the gravity of the resulting injury if she does (L) c. the burden of adequate precautions (B) liability depends on whether B<PL (b= burden, L=injury, p=probability) if B<PL, then liability attaches Bargee does not have to be aboard all the time, but has to be there at reasonable times In this case, the bargee was gone from 5 the previous evening and he still was not back when the barge broke free at 2 pm the next daythe bargee should have been aboard during the daylight hours during the short January days and during a time when there was a full tide of war activity Two potential wrong doers - tugboat defendant - shifted moorings and cut lines setting barge afloat and bargee (P's agent) - not watching barge Carroll argues that even if the tug boat broke away if the bargee was watching - P's negligence contributed to harm Law tested against the p's behavior Causal link and damages are assumed but was the P negligent? Damages of p want to shift loss on d but d saying p caused his own loss
21

Bargee should be onboard during reasonable hours could have avoided the accident altogether If p is also negligent then at the very least the d's liability will be decreased Does it matter that p might have had no foreseeability that negligent act could cause harm? Would there then still be no liability? P did have forseeability that there might be a slight possibility that an accident would occur esp considering timing - wwII during january day - busy time In Lubitz - a lot of forseeability, gravity of harm is there at some level - but no duty To impose duty would create an unreasonably high standard of care. - too much to ask for people to pick up every benign instrumentality that might cause injury from their backyards Although a golf club really is a dangerous instrumentality - what if the boy had swung a gun? Duty and reasonable care not the same thing for two reasons - Pg 132 Can be cases where there is no duty to use reasonable care - Osterling case Once a duty is established THEN jury will have to decided if the conduct was reasonable Threat of Serious Injury - if potential injury is grave the less probable the occurrence need be for d to be held negligent for not guarding against it Case: Gulf Refining Co v. Williams(SC of MS, 1938) Pg 135 D sold drum of gasoline to P's employer. Threads in drum cap were defective and when P tried to remove cap a spark set gasoline on fire resulting in P sustaining serious burns. D is liable for P's injuries due to faulty drum gasoline cap even if there was a low expectation of such an accident happening. The graveness of the injury that could be reasonably expected to occur was large enough to require the d to take extra care to prevent that risk P was severely burned when gasoline drum caught on fire unexpectedly. Pg 136 it suffices that there is some foreseeability Social Utility of the Conduct v. Cost to Avoid the Risk of Harm from the Conduct Case: Chicago, B. & QR Co. v. Krayenbuhl (SC of NE, 1902) Pg 138 P's foot amputated while playing on D's unlocked railroad turntable. D was negligent in not locking or guarding turntable and so was liable for P's injury Railroads have social utility and so utility of such a dangerous object not outweighed by injury P sustained but d's burden of locking or guarding turntable so small in comparison with injury sustained by p that to not do so was considered negligent. Advances in Technology - state of the art - d's failure to take action to prevent a certain known risk might either be negligent or non-negligent depending on whether technology exists that could reduce that risk CASE: Davison v. Snohomish County( SC of WA, 1928) Bridge in Snohomish County has an elevated road that has a poor guardrail. P's while crossing bridge lost control and went off elevated road breaking through the guardrail. D not negligent for failure to construct road barriers sufficient to keep cars on road Cars are common so county could have foreseen such an accident happening. To put up proper guardrails would place a undue burden on the county - all roads would have to be closed until guardrails were put on all roads. What must a negligence claim prove to get a jury and then by what standard (and under what kinds of evidence) does the jury decide? There has to be some foreseeability that your conduct will lead to some harm Which case has no foreseeability - Blyth? - still may have been some foreseeability at the lowest level
22

What key factor would affect the county's foreseeability? The fact that cars have been on roads for a while at this point. 2. STANDARD OF CARE Restatement (Second ) of Torts (1965) 291 - unreasonableness - if an act will result in risk of harm to another then the risk is unreasonable and the act is negligent if it outweighs any utility 292 - determining utility A - social value B - the probability that this is the best course C - the probability that there is another better way 293 - determining the magnitude of risk A - social value B - probability that it will invade another's interests C - extent of harm that may be caused D - number of people that may be effected a.The Reasonable Prudent Person Objective standard - would a reasonable person of ordinary prudence, in the position of the d have conducted his/her self as the d did, under the same circumstances Mental Attributes of the D Mental attributes not taken as part of the same circumstances CASE: Vaughan v. Menlove (Court of Common Pleas, 1837) Pg 145 D built a hay rick near P's houses. D knew that a fire might start that would spread to P's houses. D's hay rick caught on fire and P's houses were burnt down. ury found for P based on instruction of gross negligence based on standard of prudent care - New trial ordered with instructions to base D's negligence on whether he used bona fide judgment. - Ct discharges the rule D's action of not protecting his hayrick from catching fire or destroying his hayrick was grossly negligent. D knew of the high risk of a fire starting, he knew that such a fire would spread to P's house. Under the same circumstances a 'prudent' man would have exercised the correct amount of care to reduce or negate the potential risk. Insanity or Mental Deficiency Neither insanity nor mental deficiency relieves the actor from liability - must conform to the general standard of care of a reasonable person under similar external circumstances Exceptions: sudden, unforeseeable insanity or children Breunig v. American Family Ins. Co. (WI SC, 1970) - Mental Impairment and Negligence P - Breunig's car was struck by D's insured. D's insured had an insanity attack and crashed into P's car causing P's injuries. Decision: Jury returned verdict for P. D appealed. WI SC affirmed judgment in favor of P Whether D's insured insanity attack is negligent when attack occurs while insured is driving a car? Yes insured's actions could be held to be negligent but jury needed to find if she had forewarning of attack. Reason: nonliability for insanity - the attack must be such that it affects that persons ability to use ordinary care while operating vehicle or ability to control vehicle. Also attack must be sudden and unforeseen. Otherwise the insane person will be held liable.
23

Public policy reasons - innocent person should not have to bear the loss; induces estate to restrain and control insane person; discourages false insanity to avoid liability. Physical disability - taken as part of the same circumstances Blind Person Standard - blind person must conduct themselves in the way a reasonable person would act if he or she was blind: Case: Roberts v. State of Louisiana (LA COA, 1981) Pg 157 Physical impairment and negligence D's blind employee bumped into P- Roberts who fell and sustained injuries, Employee was not using his cane at the time D's blind employee acted as a reasonable prudent person would if they were blind Reason: blind man only required to take the precautions a normal person who is blind would take. Thus d's employee was acting reasonable - not necessary or reasonable to require blind employee to always carry around his cane. Since employee cannot be found to be negligent then D cannot be held liable for negligence. Duty to Investigate CASE: Delair v. McAdoo (SC of PA, 1936) Pg 148 P suing D for trespass to property and person from a car crash from D trying to pass P and his tire blowing out and hitting P's car. P sues for negligence claiming D was driving with defective tires. Witnesses testified as to the worn condition of the tire. D's negligence can be determined by a jury. Jury consists of people who are familiar with cars and know the need to maintain them to prevent dangerous risks they represent the standard of the reasonable prudent person. Jury consists of prudent car drivers who will be able to assess the standard of care used by D in driving with worn tires. Delair - blown tires - certain realities are assumed - can't get way with saying didn't know tires were bad - jury can pass judgment on everyday instrumentalities and how they might cause harm Juries - composed of different backgrounds, levels of intelligence, accomplishments and this group comes together to make a decision Distinction between duty and breach of duty - duty is the duty of the judge to consider, having looked at the p's pleadings or evidence from d. - the p bears the burden of proof on all 4 components of negligence P needs to show there is a duty and if failure to show the judge will decide no duty. - yes or no decision If there is a duty then claim goes on to trial and Breach of duty is decided by the jury Pg 147 n. 3 standard of care is a reasonable person of ordinary prudence - what is reasonability - how do we define this? What about the use of reasonable man as opposed to woman - also reasonability extended to different cultures a- a jury helps to amalgamate the abstraction of reasonability in view of the situation in question. Feasibility if there is a duty for d to prevent the risk - no in snohomish? There was no duty for the county to put up strong guardrails everywhere. - too much to ask - despite forseeability and gravity of situation In us carroll not feasible for bargee to be on board all time - feasibility is more subjective rather than duty which is more objective Custom as Evidence of Reasonable Care
24

CASE: Trimarco v. Klein (COA of NY, 1982) Pg 150 P - Trimarco's glass bathtub door broke and he was cut by the glass. He had assumed glass was safety glass but it was in fact ordinary glass. D - landlord - did not warn P that glass was not safety glass and did not attempt to replace glass. Safety glass was common at this time - many warnings issued about using regular glass in places lake bathtubs. App court decided that if no one told D of new glass then D had no knowledge and no duty of care Decision: Jury found for P and awarded $240,000. App Ct reversed and dismissed. P appealed. COA of NY reverses App decision and orders new trial. Issue: Did P show enough proof of custom (standard of care and the reasonability) and usage of safety glass to make out case of negligence on the part of D? Holding: Yes. Reason: a. When proof of an accepted practice is accompanied by evidence that the defendant conformed to it, this may establish due care b. When proof of a customary practice is coupled with a showing that it was ignored and that this departure was a proximate cause of the accident, it may serve to establish liability. c. Customary practice and usage is not universalbut if it is fairly well defined and in the same business a person either has knowledge of it or negligent ignorance d. Even if there is a common practice or usage, the jury still has to determine that it is reasonable before it is a conclusive or compelling test for negligence e. The case that the P presented was enough to sustain the verdict that the jury reached i. The jurors in this case had to determine whether or not the evidence provided in this case established a general custom or practice NOTE: The court reversed the dismissal but ordered a new trial because the trial judge had erroneously admitted certain evidence that had hurt the defense. Evidence of custom may be used to prove that the d had a duty of care Custom is not dispositive on the issue of negligence(emanuel pg 106): Case : The TJ Hooper (1932) - Learned Hand case pg 153 n. 3 - custom But custom can be used as evidence Duty and burden satisfied - But now looking at what the standard of care and how reasonable was behavior measured against the behavior an ordinary prudent man would have Reasonability determined by jury - custom can also be taken into account to evaluate reasonability But reasonable prudence is not always common prudence - jury needs to hear the evidence to determine reasonability even in view of customs In trimarco even though there may have been a custom of other landlord's installing the safety glass still have to determine how reasonable it would be for landlord to install safety glass Don't exclude evidence as to custom How to treat Custom: 1 - Admissible? - usually within reason 2 - Dispositive (decisive)? Of reasonability - not always (Pg 153 n. 3 -Reasonability never measured by custom) - behavior still has to be determined by jury against care a reasonable prudent person would use - court balances cost of administering test against cost of risk of getting glaucoma (the feasibility concern). - Pg 180 n. 4 Helling v. Carey - eye doctor still held negligent for not administering a test that was customarily not given as testified by a series of other eye doctor - helling court found negligence as a matter of law which is rare - usually this determination is left for jury to decide.
25

Emergency In cases of emergency a d will not be held to the same standard of care - standard of a reasonable person confronted with the same emergency only if the emergency wasn't caused by the d's negligence: Case: Cordas v. Peerless Transportation Co.(City Court of NY, 1941) Pg 154 D- taxicab driver - a robber jumped into his cab and pointed a gun at him telling him to drive -cab driver started driving and then observed robbed person chasing robber outside of cab. He stepped on brakes pulled on emergency and jumped out of cab which continued onto a sidewalk and struck Ps mother and 2 children. Injuries were small Whether D acted negligently by abandoning his cab when he was being accosted by a robber at gunpoint? Can a d be held liable for negligence when his actions are tempered by an emergency situation? D did not act negligently under the circumstances - an ordinary man in the same situation would have acted similarly and so such behavior on the D's part is reasonable. Kolanko v. Erie Railroad - in an emergency situation an ordinary man's judgment will be tempered and will have to act without time for thought Note: an emergency must be unforeseen, sudden and unexpected Respondeat superior case b.Child Standard of Care Children held to a reasonable prudent child of similar age, intelligence and experience unless: Child is engaged in an inherently dangerous activity, using a motorized vehicle, or engaged in an activity normally engaged in by adults then they are held to an adult standard of care. Case: Robinson v. Lindsay (WA SC, 1979) Pg 161 Children and Negligence 13 yr old pulling P-Robinson who was on an innertube while riding on D's snowmobile - P's thumb was severed by tow rope. If the minor is engaging in dangerous activities like operating a vehicle then the minor should be held to an adult standard. Minor should be held to standard of care that a reasonable prudent child of similar age would use under the same circumstances. This discourages immature individuals from engaging in inherently dangerous activities. See variation on standard rule in Robinson dealing with children Lower the standard for children - negligence based on age capacity intelligence and experience of child unless the child is engaged in inherently dangerous, motorized vehicle or mature adult activities then can be held to regular negligence standard of care **very young children under the age of 5 normally will not be held liable for negligence Look at : Dellwo v. Pearson - minor doing an adult activity - but may not include certain vehicles or guns Purtle v. Shelton - deer-hunting 17yr old - held to child standard - hunting in that jurisdiction is not a strictly adult activity Weisberg - Better standard is the adult standard - Reciprocal risk creation - if we want children to act like children then adults are implicitly held to higher standard - But when child is engaged in adult behavior then there is no reciprocity and adult not held to higher standard. Public Policy: Why a different standard for children? Standard is lowered let kids be kids - don't want to deter kids from playing A child of higher intelligence may be held to slightly higher standard - Age is key factor and then tempered with experience and intelligence
26

7-14 rebuttable presumption that children are not capable of negligence n. 4 - speaks to child's ability to figure out risk creation c.The Professional Standard of Care - Malpractice Even though professionals often will be held to a higher standard than just an ordinary prudent person still held to an objective standard of a reasonable prudent professional of that particular field would act in similar circumstances - member of the profession with the knowledge training and skill (or ability and competence) of an ordinary member of that profession in good standing For professional cases of malpractice make sure you have at least 1 very good expert witness Objective Standard for Professional Case: Heath v. Swift Wings, Inc.(NC COA, 1979) Pg 169 P's the estate of wife and son Heath suing D's - the husband Heath's estate and the airplane owners after husband crashed airplane killing all Whether court's jury instructions to hold d to a standard of care an ordinary pilot would use was misleading when they also included that ordinary person should have same experience and training as d. Holding: court's instructions were misleading concerning standard of care. - Standard of care of a prudent man remains constant but degree of care required varies according to the circumstances. Trial court included language that ordinary standard required person to have same training and experience as d Minimum standard generally applicable to all pilots Still objective because held to standard of what is common standard of care found among other members of the same profession - d's own training and experience is irrelevant Professional held to higher standard pg 169 - pg 170 n. 2 Pg 174, 175 etc all different professions that have special knowledge skills or experience are not judged by ordinary standard of care - bar is raised What is the reciprocal relationship between ordinary man and professional? Professional held to higher standard and ordinary man held to a lower standard Reciprocal risk creation may be helpful in evaluating some cases - not doctrine just an overview Roberts v. St of LS - the blind individual not held to a special standard for blind people but instead as a reasonable person would act if he were blind Have to Incorporate handicap of blind person AND the frailty of the p-old man - and weigh the reciprocal relationship Professional must act with the level of skill and learning commonly possessed by members of the profession in good standing - not normally held to guarantee that a successful result will occur Case: Hodges v. Carter(NC SC, 1954) Pg 173 D - Carter and Toppings - P's attorneys P - Hodges' drug store was burnt down in a fire. Was denied recovery by his 4 insurance companies. D's started 4 suits each against each insurance company and sent notices to Commissioner of Insurance in NC who forwarded to each company. Companies filed claims for dismissal based on improper service Eventually courts dismissed claims for improper service. P suing D for negligence in filing notices properly and then not refilling properly during the st of lim period.

27

Decision: Tr Ct entered judgment of involuntary nonsuit (matter of law - no jury) - no breach of a professional standard - but usually this breach is decide on by jury? P appealed NC SC affirms judgment. Issue: Whether a lawyer should be held as negligent when an unclear legal issue conflicts with notice filing. Holding: lawyer should not be held as negligent. Reason: Professional has to use the requisite care and have the requisite professional skills and under the circumstances where a law was unclear the d's did use the requisite care. 3 areas where an attorney's conduct is questioned: 1-Possession of knowledge/skill 2-Exercise of good judgment 3-Use of good care Specialists sometimes held to a higher standard - minimum standards of that specialty Negligence of Physician requires evidence of breach of standard of practice by expert medical testimony (except where negligence is obvious to the lay person - Ybarra v.Spangard) Case: Boyce v. Brown (AZ SC, 1938) Pg 177 D - Brown, P - Boyces - P suing D for malpractice for screw D had surgically used to fix her broken ankle Not sufficient evidence to suggest malpractice. No testimony to establish that D did not use standard and due care required by his profession. Allegation of negligence against 1st doctor - didn't use x-ray - Is failure to take an x-ray a breach of the standard of care to be used by doctors of similar situation Court found as a matter of law - didn't go to jury - have to be good reasons why taking the facts more favorably for p the d's actions were still reasonable as a matter of law. x-ray's not an ordinary standard - expert for the p's did not confirm that this was a standard but only that he himself would do it - his expert testimony of standard was too subjective - compare with Heath not that every other doctor would do x-ray but that in that jurisdiction most doctor's do take x-ray P could argue that a jury can understand that the x-ray is so commonly used to examine interior organs Relationship of expertise and custom - what do professionals usually do - need an expert to tell jury that - still even having heard an expert and the custom the jury (and sometimes the judge) still has to decide the reasonability **Only if the injury is so blatant where a lay person can identify it is expert testimony not required** possible exam question Exception Helling v. Carey Pg 180 n 4 Customary practice - professional standards themselves held to be negligent and the d liable for following them. Custom evidence of standard of care but not dispositive D can bring an expert also - uncommon to the point of almost never giving test P must bring expert Court decided as a matter of law that the standard of care was higher than the custom. Despite all of the experts agreeing what they agreed upon was not up to the standard of care. D > r and g - Cost of risk v cost of test National Standard of Care for Medical Profession Case: Morrison v. MacNamara (DC COA, 1979)Pg 181 P went to D's medical lab to take a urine analysis test. Test was administered while P was standing. P had an adverse reaction and fell hitting his head . At trial P has an expert from Michigan testify as to
28

national standard of care used when giving the urine test.- usually sitting or lying down. Tr ct rules that doctor should only be held to a local standard and not a national standard Standard of care is to be measured by the national standard. The local standard is out-dated and any major disparities between rural and urban areas has largely disappeared so no need. does't change the standard of care but does confuse what experts can be used - depending on jury Local customs can be used as evidence though Informed Consent Standard for Physicians Case: Scott v. Bradford (OK SC, 1979) P - Scott underwent a hysterectomy done by D. She then suffered from incontinence and after visiting another doctor learned she had complication caused by hysterectomy. P filed a claim for med mal stating she had not been properly informed by d of the risks of the surgery. D is negligent because he did not secure p's consent by informing her of all the risks of the operation. if a doctor has received patient's consent but has breached his or her duty to inform that consent is invalid, P states she would not have consented to the operation if she had known all of the risks - she would have to show that she would decline if she had the information. Not held to the reasonable patient would do standard only depends on what p would do. Doctor not only has to disclose what he intends to do but address the question of whether he should do it Physician standard of care is even higher than general professional standard of care - decision now is a bilateral one instead of a unilateral one controlled solely by physician Not what the reasonable physician would disclose but what the reasonable patient would want to know - this case also includes the factor that being informed would this patient refuse the procedure ***Informed Consent**** Compare with battery - no consent For negligence - no informed consent Scott - she signed up for what she got - there was consent But the doctor did not give a full understanding of risks that a reasonable patient would want to know Rules that come out of this case are Doctor has a duty to disclose the material risks (risks likely to effect the patient's decision to give consent) and potential consequences for performing or not performing the surgery With some privileged exceptions - emergency etc Pg 187 Hypo for tues - in addition to p having to prove material risk of procedure not revealed but also that had it been revealed she would have chosen a dif alternative - CAUSATION - must be proven by the p Must be an actual injury (unlike battery) A doctor treating a preg woman - woman now devout roman catholic - knows she would not get an abortion During 1st trimester diagnose with german measles - doctor declines to tell patient about risk (not life threatening to woman, but baby at high level of risk for genetic defects) Since subjective not objective analysis of whether patient would refuse based on risks - no he does not 3 exceptions to Informed Consent: Pg 187 -Common knowledge or known to patient already -Emergency -Would be detrimental to inform patient Med Mal Claims: Battery, regular negligence, informed consent, res ipsa loquitur
29

Cause of action for negligence action for lack of informed consent, the P must show: 1. the dr. failed to inform him of the material risks 2. Causation-had the dr. informed the individual patient, would the patient have chosen a different option. 3. injury- the undisclosed risk did in fact occur d.NEGLIGENCE PER SE - STATUTORY STANDARD OF CARE a-Rules of Law Case: Pokora v. Wabash (SCOTUS, 1934) Cardozo p a truck driver arrives at a rr crossing . He is unable to see if a train is coming because something is blocking his view so he inched up across tracks. His truck was hit by train. Decision: Tr Ct directed verdict for D - contributory negligence. COA affirmed. SCOTUS reverses and remands - jury needs to decide if trucker driver's conduct was reasonable and if he could have avoided train hitting his truck. Issue: Was p's behavior contributory negligence when there was common law to check Holding: p was not contributory negligent even though he did not leave his truck to check. Reasoning: It would have been uncommon, futile or dangerous for him to lave his truck . Difficult to create rules of law to govern standards of behavior - jury has to decided if conduct was reasonable within the circumstances. Duty varies with the circumstances Judge made rule of conduct - pg 201 and 203 - 'standards of prudent conduct are declared at times by courts, but they are taken over from the facts of life' b-Violation of Statute D is negligent because he violated the standard of care imposed upon him by a statute Case: Osborne v. McMasters (MN SC, 1889) Pg 204 D was a drugstore owner who sold a bottle of poison to P without labeling the bottle poison, as required by statute. P drank the poison unknowingly and died D is negligent because he violated a standard of care imposed by the statute. doesn't matter whether violation of a statute or failure to perform a duty - still can be considered negligence but By not labeling the bottle as required by the statute, d committed negligence per se - violation of the statute without an excuse. Statute in this case is a fixed standard - a duty top label poison does exist at common law Negligence Per Se 3d restatement - Prima Facie Case 1 - d violated a statute 2 - statute designed to protect against very same accident that d's conduct caused 3 - Victim part of protected class of person under statute 4 - An injury occurred Effect of Statute Negligence per se as a defense Case: Martin v. Herzog (NY COA, 1920) Pg 222 P was driving his buggy at night without lights on. This violated a NY criminal statute. P's buggy collided with D's automobile and P was killed. Unexcused omission of statutory signals was more than just evidence of negligence it was negligence per se. evidence at trial that the absence of lights was causally related to the accident. Therefore p's violation was contributory negligence so he may not recover. -negligence per se proven - then damages
30

-1st kind of jurisdiction - an unexcused violation is negligence per se -2nd kind of jurisdiction -NY - rebuttable presumption jurisdiction Buggy driver can rebutt with act of god - lightning - etc. -3rd kind of jurisdiction - just evidence jurisdiction - mere inference of negligence (circumstantial evidence) Licensing not really directed toward individual protection - and also isn't sufficient proof of not upholding standard of care Applicability of Statute to Individual - Restatement 3d part 3 Case: Stachniewicz v. Mar-Cam (OR SC, 1971) Pg 206 BAR FIGHT P was in bar when a fight broke out between other visibly intoxicated bar patrons. He was injured during the brawl and sued bar for negligence Bar can be found liable for negligence when a patron is injured during a barfight in the bar when there is a statute prohibiting bars from not eject visibly intoxicated patrons. There is a state statute OR Liquor Control act that says no bar owner shall permit loud or boisterous disorderly conduct nor permit visibly intoxicated patrons to enter or remain on property. Because p showed the bar violated the statute and that he was injured a s a result the case can be decided as a matter of law Court should still examine the appropriateness of applying standard the statute sets up. The purpose of the statute has to have been to protect the rights of a class of people that includes the p. Protection against a particular type of accident - Restatement 3d part 2 Case: Ney v. Yellow Cab (IL SC, 1954) Pg 210 - TAXI CAB STOLEN P was hit by a stolen taxi cab. P is suing taxi company based on a violation of a statute to guard against theft by not leaving keys in a parked car. violation of a statute to not leave keys in a parked car is enough to make d liable for negligence when a thief steals car and hits p. The legislative intent of the statute could be construed to have the purpose of guarding property against theft and in affect guarding against reckless driving by thieves. The legislature constructed the statute with possible foreseeability of such scenarios occurring. Therefore the jury will need to decide if the violation was a proximate cause of the p's injury and what role the 3rd party thief plays in the proximate cause chain. Violation of statute is evidence of negligence but does not create liability - evidentiary, proximate cause Dissent: violation of statute has to be the proximate cause of the resulting injury. Failure to remove the key alone could not have caused injury - the injury was a result of the thief's actions also thief was not fleeing from original location of theft. Compare with intervening cause of Watson v. Kentucky - why not a superseding criminal cause JUDGE DEFINES PURPOSE OF STATUTE Gorris v. Scott pg 209 Statute required animals to be penned separately on boats. D violated statute and P's sheep was washed overboard during a storm P cannot rely on statutory violation because the pen violation was to protect against spread of disease and not the washing of animal overboard. Doesn't fit purpose of statute. Imposition of new duty not existing at common law
31

Case: Perry v. S.N. and S.N. (TX SC, 1998) Pg 215 CHILD ABUSE Texas statute requires any person having cause to believe a child is being abused to report it to authorities. P' s 2 children were sexually abused by a Daniel Keller at day care center. The N's are suing several friends of Daniel - Perry -for negligence per se based on the Texas statute violation of not reporting abuse to authorities Decision: Tr Ct dismissed for failure to state a cause of action. COA reversed and found for P. SC reverses judgment in favor of D. Issue: Whether P can sue d for a negligence per se for not filing report under Family Code of Texas, which is considered a misdemeanor if proven) Holding: No, there is no cause of action for negligence per se Reasoning: 1 - where a statute imposes a duty that does not correspond with any duty existing at common law that weighs against the negligence per se doctrine. 2 - statute is too vague - have to determine what cause to believe is and what it means that abuse may be taking place 3 - to uphold negligence per se with this statue would broaden the scope of liability even including people whose relationship to the abuse could be extremely indirect. This criminal statute does not provide an appropriate basis for civil liability Causal connection exists But still not dispositive - no common law duty Appropriateness - judge decides the appropriateness of the statute Criminal statutes do no set up standards for civil case Judge decides if the statute is appropriate for civil causes of actions Cannot create a new duty that doesn't exist in common law and probably wouldn't Common law negligence looks at the burden - carroll towing / Osterwald B<gxf Public policy concern- duty to report any suspected abuse would impose too much burden on society Gravity in Perry is very large - but foreseeability? This statute would encompass things we would just hear about - too broad of a scope When statute is appropriate and a violation exists - negligence per se But there are exceptions Pg 221 n.4 Brown v. Shyne - Blue laws - Licensing statutes do not establish a standard of care - purpose of statute not to protect individuals from harm Excuse for Violation of a Statute - Rebuttable Presumption Case: Zeni v. Anderson(Mich SC, 1976) Pg 224 p - Zeni was walking on a pedestrian snow path, instead of snow-covered sidewalk w/ her back to traffic when she was hit by D's car. D was driving with cloudy windshield and close to curb. Statute exists that makes it unlawful for pedestrians to walk on highway when a sidewalk is available or if there is no sidewalk to walk on the left side of highway facing traffic. Issue: Whether P's violation of statute could have rendered her contributorily negligent per se Holding: No p's violation of statute does not render her contributorily negligent per se Reasoning: P introduced evidence that compliance with the statute had a greater risk of physical harm . She and others had previously slipped or fallen on the snowy sidewalk and it was safer to walk on the road. 1-Rebuttable Presumption - usual grounds for rebuttal was emergency - d has to introduce evidence of due care, allows for excuse for violating statute - 288a restatement 2d excused statute violation 2-Negligence per se - not flexible - d usually found liable as a matter of law with no room for excuses or examination of reasonable care - still a majority view
32

3-Mere Inference of Negligence - minority view that violation is only to be viewed as evidence of negligence Application of statutory standard to this case - not using negligence per se -has to be proximate cause and purpose of doctrine and also no rebuttable presumption excuse ***Statute is only a legislative standard of care and it may also have legislatively mandated excuses e.DUTY OF CARE - TO WARN, EMOTIONAL DISTRESS, UNBORN CHILDREN Why limited duty rules (and what are they or were they?)in 1-coming to aid of others 2-nied 3-wrongful: death, birth, life 4-owners and occupiers of land Duty to protect must exist in order for there to be a duty to warn No duty to warn/ no duty to rescue - baseline Some exceptions! Special relationship - between d and p or d and third party Elevated standard for common carriers and innkeepers Responsibility towards third persons based on their children (Lubitz) does not exist - Unless it's a child with known vicious propensity - and injuries are caused by that known risk 1-FAILURE TO ACT Affirmative duties to act a. No duty to aid of others in peril even if you could aid the person at no risk to yourself. b. If you assume the duty to act, you must exercise reasonable care Exceptions where have a duty to act: a. If the peril is due to your own negligence b. Special relationships between P and D. is not dispositive, just will enhance the courts ability to determine whether a duty exists. i. Parent-kids (parents not held liable 1st time kid does something) ii. Common carriers/Innkeepers iii. Shopkeepers/Invitees Maybe: -schools /student -employer/employee Look at b<pl burden c. Special relationships between D and 3rd persons- Has to do with the ability to control and predict the behavior of the 3rd person who may harm. Defendant-third party relationship CASE: Tarasoff v. Regents of Univ of Cal (SC of Cal, 1976) pg 432 Psychiatrist employed by d learned from a patent that he intended to kill p who had spurned his advances. Psych alerted the campus police who detained podar for a short time. He and other psych decided not to do anything further. 2 months later podar killed tarasoff. Special relationship between d doctors and patient who killed p the doctors can be held liable due to a duty to warn p of patient's intentions if a reasonable person would have done so
33

Doctors don't have a duty to confine but they do have a duty to warn because they have duty to control patient and prevent patient from harming the p Tell others, not because of a special relationship with the other person, but bc of a special relationship b/t the psych and the potential injurer (poddar) Can you really even confidentially foresee that patient will carry out their intention? Is it a foreseeable targeted risk? Ethically - need to reveal a client's threat even when under the confidentiality - look at ethical rules within jurisdiction pg 436 n.2 catch 22 situation - could also be held liable when you do reveal confidential information Foreseeability, special relationship, and undertaking b/c of job. Professional obligation of shrinks: balance factors: o Ability to control the outpatient o Public interest in safety from violent assault o Difficulty in forecasting whether the patient is a substantial risk of physical harm to others o Goal of placing a mental patient in the least restrictive environment as possible o Safeguarding the confidentiality of the relationship 2-NEGLIGENT INFLICTION OF EMOTIONAL DISTRESS Physical injury without physical impact CASE: Daley v. LaCroix (SC of Mich, 1970) Pg 450 d driving near p's farm, goes off the road and flies through the air hitting a utility pole, causing an explosion which also resulted in property damage. p's claim they suffered emotional distress - and lost weight. a jury can reasonably find a causal relationship between fright by d's negligence and injuries of emotional distress when there was no physical impact. where emotional distress prox caused by d's negligence results in a definite and objective physical manifestation of injury the p may recover, even if there is no physical impact at the time of the frightening event. Dissent: opens up courts to many claims of emo distress. p's in this case did not suffer definite and objective injury, they suffered indefinite and subjective injury - should have fell within the general rule and recovery denied Physical manifestations - outgrowths of emotional distress Under the old rules Needed physical contact to allow for emotional distress - physical impact** -why have this impact limitation? - to keep people from bringing in fraudulent claims, brings credibility to a claim with physical impact -impact - makes the injury of emotional distress more objective and definite Pg 454 n.2 Daley does away with impact rule but not the physical manifestation of emotional distress (an after affect of the incident, symptoms proven by a doctor) Dissent upset - found the evidence of the physical manifestation to be insufficient to show an after affect of the incident The requirement for intentional infliction of emo distr - also did away with physical impact- siliznoff - p only threw up but court felt that was enough for a cause of action Presence at scene plus close relation to injured CASE: Thing v. La Chusa (SC of Cal, 1989) pg 456
34

john thing was d truck by d's car. P mother neither saw nor heard accient, she was told of john's injury by a daughter who had seen the accident. P rushed to the scene of the accident and saw her injured child, believed him to be dead. Sued d for emotional distress DECISION: tr ct denied recovery to p. coa reversed summary judgment. SC judgment reversed - p cannot recover. ISSUE: can a p not in the zone of danger who learned of son's injury after event hold d liable for emotional distress HOLDING: no REASONING: Amaya - limited to p's in the zone of danger Dillon v. Legg - young girl hit by d's car mother and sister present, sister was close to scene(in the zone of danger) and mother was further away(outside of the zone of danger) artificial requirement of being a reasonably foreseeable p (closely related, - court allowed mother to recover as well - held that the zone of danger would be absurd to apply in such a case Molien - patient with syphilis(wrongly diag) - warned husband - emotional distress upheld (physical harm, accident, sudden occurrence eliminated) - seems crazy to require doctors to warn and then to hold them liable for false warnings!! Compare with tarasoff P not in zone of danger - limitless liability -can be extended to anyone who is a foreseeable p Court decides should be limited by special relationship and 3 factors to consider: 1-closely related to injured victim 2-p must have been present at the scene and aware that event was causing injury 3-must have suffered emotional distress beyond that to be expected of an unrelated by-stander and is not abnormal response to circumstances P did not satisfy any of the new factors Pg 463 n.4 NY - zone of danger applies 3-WRONGFUL DEATH, BIRTH, LIFE UNBORN CHILDREN No duty to an unborn child Mother can recover for loss of child and subsequent distress but action in the name of the infant cannot be brought What if stillborn? Still no duty - requirement that child be born alive CASE: Endresz v. Freidberg(NYCOA, 1969) pg 464 p was 7 months pregnant when hit by d's car. 2 days after accidnet she gave birth to stillborn twins. 4 actions brought, her, the father and the twins. Twins suit dismissed d whose negligence resulted in the stillbirth of p's does not owe those stillborn p's a duty of care and will not be held liable for wrongful death. If the wrongful death statute of the state does not apply then, no there is no duty to persons unborn. Dissent: fact that one might die right after birth and can recover, but one that dies right before birth cannot is not fair Pg 465 - wrongful death statute of ny - contains word 'decedent' - didn't really die because they weren't born-still born twins can't be decedent - in order to die have to have been born -pure question of looking at the ny wrongful death statute' - in derogation of the common law (getting around the common law)- common law no cause of action can be brought by a dead person - But now with statute even a unborn child who dies from a negligent action cannot bring an action
35

Unborn viable fetus injured in tort- when born can bring an action (even if only had 1 second of viable life - ancient idea was that you had to hear the baby's first cry) Pg 466 - justifying the statute - parent's action on behalf of the stillborn - if fetus killed during a viable state would create a windfall for mother (double jackpot) - damages - mother would be compensated for distress - to give damages for a life in utero have to assess the value of a life in utero(not worth very much?) Exam question - what if mother was driving to an abortion clinic? Congenital defect - Wrongful Life Claim CASE: Procanik by Procanik v. Cillo (SC of NJ, 1984) Pg 469 p through mother claims d doctor's failed to diagnose his mother with german measlesand p was born with congenital rubella syndrome suing for pain and suffering and his parent's impaired capacity to deal with his defects and future expenses related to defect DECISION: tr ct dismissed (failure to state a cause of action), sc- p may recover future med expense, but not for pain and suffering and impaired childhood ISSUE: can p bring a cause of action against doctors for misdiagnosing mother's german measles and suffering congenital defects as a result HOLDING: p can recover medical expenses related to congenital defects REASONING: Gleitman case could not assess the value of p's life if he had not been born with impairment - reluctant to take into account mother's right to terminate, no recovery for emotional distress Roe v wade - started new strain Berman - emotional distress awarded but not medical expenses(would give parents a windfall) No cause of action for child Procanik - doctors owed a duty of care to child But only medical expenses incurred by parents are predictable, certain and recoverable Child bring cause of action because the parents missed the statute of limitations Court cannot give impaired life award because too hard to put a value on 'we seek only you respond to the call of the living for help in bearing the burden of their affliction.' Flaw in argument about diminished childhood - if they would have used info to abort then how can they be requesting money for his life? Wrongful Life Parents bring action on behalf of the child -Usually parents bring action directly as wrongful birth but they missed the sol Kids bring wrongful life and parent's bring wrongful birth Wrongful life claim - I would be better off dead -if the mother had been a devout roman catholic doctor could have chosen not to inform of german measles knowing she would have the child anyway But this is not an informed consent case Doctor not responsible for the mother having german measles - he did not injure the child directly The originating conditions were not the fault of the doctor So why is infant suing doctor? If not for the doctor's negligent advice the mother could have chosen to abort and the child would not have been born
36

Gleitman Pg 471 - impossible to measure the value of life with impairment against the the nonexistence of life itself Asking damages for pain and suffering for negligent infliction of emo dist And special expenses (medical costs) (different jurisdictions go different ways on each of these claims and each subcomponent of those claims - necessary to know each of the subcomponents of such claims) Wrongful birth - brought by parents - nied and special expenses Wrongful life - brought by anyone Wrongful death Seems like artificial rules are being set up to restrict liability - doctor does have a duty to a baby in utero - a viable child can bring a cause of action for an in utero injury - not only doctors can be held liable - if the child in Endresz had survived could have brought an action against negligent driver Why is court finding it difficult to accept the child's claim of emotional distress - they feel that some life is better than no life If this was put into the hands of the jury how could the jury figure out the amount to give for emotional distress -what about a more cognizant adult bringing a wrongful life suit - against parents or institutions usually not pg 478n.6 Compare to right to die claims pg 476n.1A - reverse damage awards 4-OWNERS AND TRESPASSERS, Rejection a.Outside the Premises Landowner has a duty to prevent unreasonable risk of harm to persons off the land from artificial conditions on the land - exception for trees CASE: Taylor v. Olsen (SC Ore, 1978) Pg 480 p injured when she struck a tree field form d's property on a dark and windy night d was logging and had recently cut down half of the trees on the land - tree was decayed but the decay did not extend through the bark. d cannot be held liable for injury sustained by p from a felled tree from his property obstructing a highway when the d had no knowledge of the state of tree. Jury must determine reasonableness of d's tree being in the road. Also need information of notice d had or should have had concerning the state of the tree Landowner must exercise reasonable care to prevent an unreasonable risk of harm Although d was logging on land and should have made a reasonable inspection of the land - the state of the tree gave no evidence of it's decayed condition - directed verdict is affirmed Compare to Lubitz - b<pl Unreasonable for landowner to have known because the tree decay did not show through the bark - he would have had to cut or bore into the tree, unforeseeable Artificial v. Natural CASE: Salevan v. Wilmington Park, Inc. (Sup Ct to DE, 1950) Pg 482 p hit in the back with a baseball while walking on road adjacent to d's baseball park
37

d has a duty to exercise reasonable care to prevent injury to travellers lawfully using highways adjacent to their property? the game of baseball is inherently dangerous, also historically balls were hit outside of park - d has duty to use ordinary reasonable care and to take reasonable precautions, such as a higher fence Ballpark owner should have taken more precautions - like having a higher fence - but in this case the landowner had created an artificial structure and also knew that foul balls often flew outside of ballpark What standard of care required - that of an ordinary person Pg 483 n1 and 2 natural v. artificial conditions Pg 484 n3 - violation of statute or regs - ordinances - duty to the municipality and not an individual but when a person purposely builds something like a snowman or a snowmound may be held liable to an individual b.On the Premises (A) Trespassers No duty owed to trespassers to make land safe or to warn or to protect the trespasser in any other way except for children - owe children a duty of reasonable care under certain conditions i. No duty is owed to undiscovered trespassers ii. Duty to Discovered or anticipated trespassers: 1. warn of or make safe concealed, unsafe, artificial conditions known to the landowner involving risk of death or serious bodily harm 2. use reasonable care in the exercise of active operations on the property iii. Attractive Nuisance Doctrine-duty on landowner to exercise ordinary care to avoid a reasonably foreseeable risk of harm to children caused by artificial conditions on his property. P must show: 1. dangerous condition that the owner is or should be aware of 2. knows or should know children frequent vicinity of condition 3. likely to cause injury b/c kids inability to appreciate the risk 4. costs little to prevent compared to the magnitude of the risk Undiscovered Trespasser CASE: Sheehan v. St Paul & Duluth (US COA7th, 1896) Pg 485 p walking on d's railroad tracks, his foot slipped and got caught between rail and cattleguard was hit by train - p sued P cannot recover for injuries sustained while on d's property when p is an unknown trespasser and d owes no duty. Not foreseeable that p would walk onto d's railroad tracks - p was not a passenger or around an area passengers could be expected - p assumed the risk, d had no notice - no breach of positive duty. exceptions for traps, spring guns, children, willful or wanton behavior on the part of the d(duty to use ordinary care or to warn) And obviously dangerous conditions Frequent trespassers? Landowner should anticipate trespassers and exercise reasonable care to protect them Tolerated intruders - amounts to permission (B) Licensees Duty to Licensees-enters onto the land with permission for own purposes or business, rather than for the possessors benefit. SOCIAL GUESTS i. warn of dangerous conditions-natural and artificial-known to the owner that create an unforeseeable risk of harm to the licensee that he is unlikely to discover ii. exercise reasonable care in the conduct of active operations on the property
38

CASE: Barmore v. Elmore (App Ct of Ill, 1980) Pg 489 p went to d's house for a mason lodge meeting. d's son threatened p, and followed p out of house with a steak knife where he then stabbed p P sued claiming the d was negligent in protecting him from a dangerous condition on his property (son with a mental illness) d's duty depends on whether p was an invitee or a licensee dist ct directed verdict for the d. p appealed, app ct affirms D cannot be held liable for attack by a third party on p when p is a licensee Invitee - enter d's land to further the owner's business Invitee does not have to gain from entering owner's property A licensee - enters owner's property for their own pleasure or benefit (social guest) Greater duty is owed to an invitee, Licensee usually has to take the premises as he finds them Duty towards licensees can be extended to protect from third party criminal act Court finds p to have been a licensee - d should have warned about danger if they had knowledge of it but Court finds no evidence of d's knowing danger, also p had had previous contact with d's son without incident Bare licensees - sales persons, canvassers, social guests Artificial rules set up, but if father had scienter(actual knowledge or should have known) then maybe a duty would arise on pure foreseeability grounds - Pg 432 n. 3 Father could argue that there was a long time lapse between incident and last violent act of son - so not foreseeable (compare to Blythe) also that the son was 47 years old (but this person could be labeled insane and then father would be guardian - compare to McGuire v. Almy pg 25 How could father have been found liable - if the plaintiff had been an invitee Invitee - furthering the landowner - a business visitor But he was there for fraternity business - which didn't benefit the d but did benefit in part the p himself (although fraternity benefits both p and d - could have been defined as an invitee in another jurisdiction) - he was then a social guest - a licensee - the only duty that exists is the duty of d to warn or protect from hidden dangers that are known to d, but otherwise the p has to take the premises as he finds them -passive v active conditions - if active most jurisdictions devolved a duty to warn (C) Invitees Duty to Invitees-enter the land by invitation of the landowner: either for a purpose connected with the business of the landowner or as members of the public for which the land is held open to the public i. Same duty as licensee AND ii. Make reasonable inspections to discover non-obvious dangerous conditions and make them safe. iii. Duty to rescue if injured on instrumentality 1. invitee - duty to take affirmative steps and improve the condition and make sure it is safe for pp who enter onto property. One loses invitee status if you exceed the scope of the invitation o Whelan v Van Natta-In front of store was invitee. In the back, becomes a licensee b/c place where he went was not a place where people usually go. o Campbell v Weathers-Moves from one part of premises to another to use the bathroom. Here status didnt change b/c frequent customer and bathroom open to the public. CASE: Campbell v. Weathers (SCt KS,1941) Pg 492 p entered d's lunch store and without purchasing anything went to use the bathroom, p fell through trap door in the floor of the bathroom
39

D can be held liable for injuries to p while p in store as an invitee - P was a common customer, so was an invitee CASE: Whelan v. Van Natta (Court of Appeals of KY, 1964) Pg 495 p entered d's store bought cigs. Asked d about a box for his sn, d directed him to go to backroom and get it. P entered room that was dark and fell down a stairwell. D claims he did not know that the room was not lit D cannot be held liable if p's status changes from licensee to invitee before he is harmed on the premises. P is a licensee while on the premises within the scope of his invitation, P went outside the area of his invitation, Court held p to be a licensee Policy of extending highest level of duty to invitees? - encourages vendors to maintain a reasonable care of premises where a potential customer will be to avoid them being harmed As the p shifted from buying cigarettes (he completed the transaction that incurred a benefit on the d) to getting a box for his son (transaction for p's benefit) Pg 495 n.5 Invitee even though no mutual benefit Bare licensee - owed very low duty - duty of landowner to avoid willful or wanton misconduct (applies to door to door salesmen - not really invited on premises) (E) Rejection or Merging of Categories -Modern trend rejects distinctions and apply a reasonable person standard to dangerous conditions on the land. -CA breaks down all the distinctions and says duty to warn licensee about known danger and goes so far as to say that a known trespasser should be warned too! CASE: Rowland v. Christian (SC Cal, 1968) Pg 502 P was a social guest in d's apartment. Went to use the bathroom where he then severly cut his hand on the broken faucet. D did not warn about broken faucet but had known of it for 2 weeks and had complained to building manager. D can be held liable for injuries p sustained while a social guest(licensee) in d's house and historically d owes no duty of care to a licensee. Categories should only be used as evidence and should not be dispositive. Even though p was a licensee at the time of injury such status is not dispositive of the duty of care d owes him. Instead the test is whether the landowner acted as a reasonable person The common law approach of attributing liability based on the category of licensee, trespasser, or invitee does not take into account factors like: -causal connection bt d's actions and p's injuries -public policy of discouraging future harm -availability of insurance Foreseeability may vary even within the categories P was aware of faucet's dangerous condition, that p did not know of danger and would come in contact with it, and she did not warn - p was negligent Dissent: getting rid of the categories does not help establish what duty is owed if any Could place too much burden on a land/home owner - could give rise to unlimited liability California establishes that landowner has a duty to warn not only licensees but also invitees and trespassers - ordinary negligence standard applies (Occupiers Liability implemented in England overturned the distinctions - standard of 'common humanity' pg 506 n.2)
40

352 ne2d 368 - Miller case in NY Exception for obvious dangers Pg 497 n.6 (invitee should be aware on their own) 2-BREACH OF DUTY PROOF OF NEGLIGENCE - Proof of Breach of Duty (A) Direct Evidence (B) Court and Jury: Circumstantial Evidence Circumstantial evidence - evidence of one from which the evidence of the fact to be determined may reasonably be inferred. Witness testimonies that can lead to conclusion being drawn Importance of careful proof Case: Goddard v. Boston & Maine RR Co. (Mass SC, 1901)BANANA PEEL Pg 229 P exited train and slipped on banana peel on platform. Many passengers on platform D cannot be held liable for negligence for banana peel on platform causing P's injuries. P did not present enough evidence that d had not taken reasonable care to prevent banana peel from being on platform - likely that the peel had just been dropped by a person who was just leaving train also. Unreasonable to make it a duty. Unreasonable burden on railroad for things that may have just happened that they wouldn't know about Case: Anjou v. Boston Elevated Railway (Mass SC, 1911) Pg 230 DIRTY BANANA Facts: P Anjou slipped on banana peel on D's platform - She had exited train and waited for crowd to dissipate. Banana peel was gritty, dirty flattened and all blackened. Duty of D to clean platform periodically to remove things that could endanger people on platform D is liable for banana peel on platform causing P's injuries when P presents circumstantial evidence that D did not use reasonable care to remove banana peel. P presented enough circumstantial evidence to link the presence of the banana peel with lack of reasonable care - witnesses testified to the state of the banana peel to conclude that it had been there for a while - long enough for the D's employees to have seen and removed it LEGAL LIABILITY Constructive Notice Case: Joye v. Great Atlantic and Pacific Tea Co. (US COA 4th, 1968) Pg 231 SUPERMARKET FALL P - Joye slipped on banana peel in supermarket -D D is not liable for P's injuries due to negligence when there isn't sufficient circumstantial evidence P did not present enough circumstantial evidence: 1 - no evidence that D's store put peel there 2 - D's store had no actual notice that peel was there P presented no evidence to establish how long peel had been on floor Case: Ortega v. Kmart Corp.(Cal SC, 2001) pg 231 SPILT MILK Constructive Notice Doctrine - store owner should make reasonably frequent inspections of premises to make sure no unsafe conditions exist. If a p can show unsafe condition existed for longer than max reasonable interval between inspections there can be enough for jury to conclude d was 'on constructive notice'. P Ortega slips on spilt milk in D's store. P did not know how long milk was on floor and did not present evidence to determine how long it had been there. D store manager testified that inspections were
41

made every 15-30 mins but milk could have been there for up to 2 hours and also stated that management did not know if aisle was inspected at all p provided enough circumstantial evidence that d had constructive notice of spill and enough time to use reasonable care to prevent spill causing p's injuries. p has burden of proving that owner had notice of dangerous condition and had enough time to use reasonable care. Burden stays with p throughout P can satisfy burden with circumstantial evidence P needs to provide this evidence and jury needs to decide- which is what happened- jury determined that spill was on floor long enough for owner to have had constructive notice Foreseeable or Continuous Dangerous Condition Case: Jasko v. Woolworth (Col SC, 1972) Pg 233 PIZZA FALL P - Jasko slipped on pizza on floor of D's store near a pizza counter Does P does not need to show evidence that D had constructive notice when the injurious situation was recognizably dangerous - reasonable probability that food would drop on floor? P needed to prove that D's pizza counter was reasonably recognizable as dangerous and so no constructive notice was necessary since dangerous conditions were foreseeable or continuous Must be some Evidence of Negligence - by causing the dangerous condition or failing to remedy it Case: H.E. Butt v. Resendez (Tex SC, 19999) Pg 234 GRAPES FALL P slipped on grapes in front of D's store display of grapes- with loose grapes for sampling Display had floor mats in front of it and the floor was non-skid. Sampling display was on a level table with railing - and there were warning cones d's display was not an unreasonable risk of harm P did not show: 1 - the store had constructive notice of conditions 2 - the condition was an unreasonable harm 3 - store did not use reasonable care to reduce or eliminate risk 4 - store's failure to use care was proximate cause of p's injuries Has to be some evidence of negligence D can then prove that they did use reasonable care P has three burdens of proof on the issue of negligence 1-burden of pleading - allege sufficient facts in the complaint 2-burden of coming forward with enough evidence to avoid directed verdict - convince judge that reasonable jurors could find on a more-probable than not basus that p's contention is correct 3-burden of persuading jury JUDGE decides questions of law: DIRECTED VERDICT -STATE OF FACTS - reasonable people could not differ - instruct jury how they will HAVE to find -EXISTENCE OF DUTY JURY is the fact finder - if reasonable people could differ on the facts -What happened -And what was the particular standard of care - was it breached and did it proximately cause p's injuries
42

(C)RES IPSA LOQUITUR - the thing speaks for itself - the fact that a particular injury occurred in of itself establishes a breach of a duty. Case: Byrne v. Boadle (Court of Exchequer, 1863) Pg 237 BARREL OF FLOUR P walking down street in front of D's shop when barrel of flour falls on him Decision: Jury found for D- no proof of negligence on part of D or D's employees. Court of Excheqeur enters verdict for P. D is liable for negligence of flour barrel falling from his store even if P presented no evidence to connect the act with D. store has a duty to take reasonable care that their products are in their control. No affirmative showing of d's actual carelessness - breach of duty Prima facie case of evidence if the instrument that obviously was in control of d causes an injury directly P does not have to show any further evidence other than the barrel could only have been in d's control, and does not have to show that it could not have fallen without negligence. This shifts burden of proof to D to show that the barrel was NOT in their control or there was a reasonable excuse. At least three requirements for res ipsa loquitur: 1 - no direct evidence of d's conduct - no need for the doctrine if court knows the details of d's behavior and thus can establish if there is causation - p has to be in the dark 2 - seldom occurs w/o negligence - most of the time such an injurious event occurs it was caused by someone's negligence 3 - in defendant's control 4 - rule out p's contribution to the injury (causation and comparative negligence) Also sometimes 5 - accessibility of information - explanation of events is more readily available to d than to p Inference of someone's negligence - more than 50% likely only caused by negligence CASE: McDougald v. Perry (FL SC, 1998) Pg 240 SPARE TIRE P McDougald suing truck driver Perry - D for accident where a spare tire fell from Perry's truck and crashed though p's windshield injuring the p. Perry said the tire was secured to truck's underside with a chain and he inspected the truck before the trip The tire accident is a type of accident that would not happen but for a failure to exercise reasonable care by the person who had control of the tire - based on common experience and general knowledge. REASONING: District Ct: Reversible error for judge to instruct jury on res ipsa loquitur Res ipsa permits but does not compel an inference of negligence - just a rule of evidence Still need some proof Injury had to be under direct control of d - accident would not occur without negligence BUT SC finds that: To suffice res ipsa loquitur P must show that most of the time negligence is the cause (type of event) And P must still present some evidence of its 'immediate, precipitating cause.' Concurrence : notes Byrne v. Boadle - common law basis of res ipsa loquitur survives Res ipsa - general experience of jury - doesn't need expert testimony usually - however for med mal still may need expert testimony to setup if it is a res ipsa loquite r situation P does not have to show that there are no other explanations- just a 50%+ probability
43

Res ipsa loquitur is rebuttable SHOWING THE NEGLIGENCE WAS DEFENDANT'S P must show by a preponderance of evidence that the negligence was probably that of the defendant CASE: Larson v. St. Francis Hotel (Cal D. COA, 1948) Pg 247 P celebrating Victory over Japan day on san francisco streets when armchair falls from somewhere and knocks her unconscious. No one observed chair falling out of St Francis Hotel but it was assumed. Not a res ipsa loquitur situation because the p could not establish that the inherently negligent occurrence was connected directly to the d. P must prove: 1 - there was an accident 2-instrument that caused injuries was in exclusive control/management of d 3-if d had used reasonable care the accident would not have occurred Guests in this situation could have some control To hold hotel negligent would mean that to prevent guests from doing such things they would have to post guards at every room - not such a usually occurrence that this would be a necessary cost. Now - more than likely in d's control - not necessarily exclusively in d's control Multiple Defendants If the p can demonstrate the probability that the injury was caused by negligence of at least one of the defendants, but cannot show which one, the doctrine can be applied against all. Joint and several liability - all held liable for an amount and figure out among themselves what each owes CASE: Ybarra v. Spangard (Cal SC, 1944) Pg 250 Shoulder paralyzed P went to hospital for appendectomy, Sustained serious injury to the shoulder after surgery Sues attending physician, hospital owner and anesthesiologist D can sue multiple parties for res ipsa loquitur negligence if he knows at least one of the defendants was negligent but cannot pinpoint which one. multiple parties can be held negligent res ipsa if there is probability that at least one was negligent Tr ct argued that p has not shown instrument that caused injury was under d control because he has not shown which instrument caused his injury But sc says how could he have known if he was unconscious during the surgery - more than likely that one of the parties was negligent and all had control of his body - duty of care - there is an inference of negligence still that the d's have to overcome Tr ct without a jury found against d's who testified that they knew of nothing that could have caused injury Joint and several liability This involved a professional standard of care - superior knowledge For strangers not as high a duty Res ipsa - cannot be any inference of contributory negligence Res ipsa is used as circumstantial evidence - can be used by p to meet burden of proof - but still up to jury to decide if res ipsa loquitur proves negligence on part of d Affirmative evidence submitted by p - but in res ipsa cases most times p in the dark (passive p) What about airplane crashes - res ipsa might be there for p- family members of deceased - but not sure CASE: Sullivan v. Crabtree (Ten COA, 1953) pg 254
44

FACTS: P deceased son was killed as a passenger of truck which swerved off highway and crashed. p sued truck driver and owner HOLDING: no p did not satisfy burden of proof - d was able to show by a preponderance of evidence that he was not negligent REASONING: Three effects of res ipsa loquitur: 1-warrants an inference of negligence 2-raises a presumption of negligence 3-can also shift ultimate burden of proof to d to prove that more likely than not he was not negligent alternative cause Reasonable evidence in the absence of explanation from the defendant of negligence P did not satisfy their burden of proof that negligence was res ipsa - jury decided If only using res ipsa as an inference- then d doesn't have to rebut anything Jury will look at facts Effect of Res Ipsa: (1) no directed verdict for the D because the P has made a prima facie case (2) D may still prove due care and the jury could find for him. 3-CAUSATION A.CAUSE IN FACT Sine Qua Non - But for d's negligent act p would not have been injured CASE: Perkins v. Texas and New Orleans Ry. Co. (Louis SC, 1962) Pg 259 P's car hit by D's train. train was 12mph over speed limit. Train and car could not see each other Even if the train had been at speed limit could still have been a crash - warning was too short for the train to be able to brake in time. Excess speed was not a substantial factor In equal fault the position of the d is stronger P failed to discharge burden of proof Pg 262 n. 8 Proof of Causation - Substantial Factor Demonstrate actual causation by a preponderance of evidence - prove the probability that injury would not have occurred but for some act on the part of the defendant Substantial factor -> more likely than not Difference between possibility - something could happen less than 50% or 50% Probable - more likely than not to happen 50%+ Mentioned in Gulf Refining Pg 136 didn't have to prove injury was probable only that it was possible this element was foreseeability (possible not probable) ***CASE: Reynolds v. Texas & Pac. Ry. Co. (Louis COA, 1885) Pg 262 STAIRCASE FALL P injured when she fell down train stairway, which was not lit and had no handrail P established that d's failure to light the staircase was the actual cause of her injury when P might have fallen when if the staircase had been lit. P adequately established actual cause - actions of the d to not light the staircase greatly multiplied the chances of an accident Possibility that injury might have occurred regardless is not enough to break the chain of causation Causal link between darkness of stairwell and p falling Post hoc ergo Proctor hoc is a fallacy - but Negligence in this case - lack of handrail and no light
45

D claims even in broad daylight woman could have fallen anyway But d's negligent actions greatly enhanced possibility - becomes more likely than not that act would cause injury Pg 268 n1 Zuchowicz Dosages in medical malpractice cases -when a negative side effect is demonstrated to be the result of a drug and the drug was wrongly prescribed in an unapproved and excessive dosage then the p who is injured has generally shown enough to permit the fact finder to conduce that the excessive dosage was a substantial factor in producing the harm -Some evidence is required A mere suspicion that the d's negligence caused the p's injuries will not suffice CASE: Gentry v. Douglas Hereford Ranch, Inc. (Mon SC, 1998) Pg 263 Staircase Trip w/ Rifle D1 (Bacon) visited his grandmother-in-law's(D2) ranch While carrying a loaded rifle he tripped on deck stairs, rifle went off and shot P (a guest at ranch). P died. P's estate suing both D1 and D2. D1 filed bankruptcy. D2 - claim proceeds - negligent maintenance was cause in fact of the accident A suspicion, regardless of how particularized it may be is not sufficient to sustain an action or defeat a motion of summary judgment - P did not offer any substantial evidence that any condition of the property caused D1 to stumble - P has not proved a cause in action Expert Testimony CASE: Kramer Service, Inc. v. Wilkins (Miss SC, 1939) Pg 267 SKIN CANCER FACTS: P - guest of D's hotel is cut on the forehead by a piece of glass falling from a broken transom. Cut doesn't heal - 2 yrs later skin cancer develops at point of injury 2 medical experts testify that 1-there is remote possibility that cut could have caused cancer and 2 that there is no causal connection between injury and cancer Mere possibility that cancer developed from cut does not provide the requisite causal connection. Jury should not have been permitted to find for p. Causes of cancer are outside the knowledge/experience of layman - the medical testimony made it highly unlikely that cut caused cancer. Necessity of expert witness to establish evidence of probability that d's act caused injury when laymen have no appreciable knowledge of injury Increased risk, followed by actual damage, to show evidence of a substantial factor for causation CASE: Herskovits v. Group Health Cooperative of Puget Sound (Wash SC, 1983) Pg 270 Medical Misdiagnosis P consults a group of doctors - Ds. Ds don't diagnose P for lung cancer. P is diagnosed later and dies. Had Ds promptly diagnosed P he would have had a 39% chance of survival. After delay his chances dropped to 25%. Ds argue that P would have died anyway so their negligent delay in diagnosing was not cause in fact of Ps death. P argues that delay proximately caused 14% reduction in chances for survival. a P with 39% chance of survival has a cause of action against d's who negligently misdiagnosed and reduced chances by 14% p'schance of survival being decreased by 14% is sufficient evidence to allow jury to decide if the reduction was a cause in fact of death - Court also limits any recovery to direct items of damage due to premature death(lost earnings, additional med expenses, but not for emotional suffering.
46

Vary the rule when it comes to certain kinds of med misdiagnosis - in some circumstances p may be able to recover even if can't prove more likely than not causal link Hypo Can't prove but for probability Lost opportunity due to misdiagnosis - compensation for up to 40% of value of victim's life had he survived -how do you value a human life Rule to judge whether certain scientific testimony is admissible CASE: Daubert v. Merrell Dow Pharmaceuticals, Inc. (USCOA 9th, 1995) BENDECTIN New federal "scientific knowledge" standard Evidence does not need to be generally accepted Must be derived by scientific methods (capable of being tested) FACTS: P's (children born with birth defects) sued D - Merrell Dow - claiming use of Merrell Dow's drug Bendectin by their mothers caused their birth defects. Ps brought in 8 experts - they testified a reanalysis of previously published epidemiological studies and thei own unpublished works, suggested causal connection between bendectin and birth defects. D countered that none of the research concluded that drug could cause defects. Moved for summary judgment Expert witness testimony two prong standard of good science and fit. scotus sets new standard Old standard Frye only generally accepted scientific theories could be presented to the jury Now two parts to consider with expert testimony Whether testimony is derived from a scientific method and is good science -whether theory can and has been tested Testimony must also be shown to be relevant to the issue In Daubert COA applies standard of Rule of Evidence 702 1 - good science -expert's bald validity would not be enough, has to show some objective, independent validation of the expert's methodology a. Show that testimony grows out of pre-litigation research b. Or that the research has been subjected to peer review or has been published and received favorably 2-fit - does testimony fit issue - did testimony establish any causation? 1. Direct proof 2. Epidemiological proof that it more likely than not was the cause of the birth defects i. Experts could only say that it somewhat increased likelihood of defects - but it would have to state that it more than doubled chances and ps witnesses could not give that quantitative testimony. Deus ex machina - an outside force intervenes to help with a seemingly unsolvable problem in an unlikely way Eleemosynary - philanthropic Concurrent causes Multiple Negligence - But For variation D cannot claim they are not an actual cause of p's injuries simply because come other person's negligence also contributed to the harm (joint tortfeasors) CASE: Hill v. Edmonds (SC NY, App, 1966) Pg 282 tractor Trailer Crash
47

P passenger in D1's car (car driver). D2 - tractor trailer owner left tt in the middle of road without lights in a storm. D1 crashed into trailer from behind D1 saw truck four car lengths in front of her and tried to swerve D2 can be held jointly liable for accident when D1's negligent actions contributed also d2's liability cannot be dismissed because accident would not have happened without d1's actions both negligent acts contributed to the accident and so both have to share the responsibility for the entire result Variations on the 'but for' test Absence of crucial causal link - characterized usually as the 'but for' test The judge is the gatekeeper on all 4 elements of negligence - preliminary determination on whether the but for test has been satisfied Substantial Factor Variation Two events occur concurrently to cause harm - but independently they would have been sufficient enough to cause the harm they jointly caused If a substantial factor then cause in fact CASE: Anderson v. Minneapolis (SC Minn, 1920) Pg 283 Forest Fire D negligently causes a forest fire. His fire combines with another fire and burns Ps property. d can be held liable for harm caused by his negligently starting a fire when another concurrent fire caused damage and would have caused damage without his own negligent fire. d's fire was still a material and substantive factor in p's house burning down and so d is liable for entire damage regardless of other independent fire. But for test variation - multiple parties, 1 unknown fire of unknown origin and fire of d's negligent action In order to find d negligent for an injury caused by that d and another unknown source have to vary the 'but for' test -use of Cook v. Minn. Case Make sure it was understood correctly - no liability when involvement in neg of an unknown source This court disagrees - d is going to be liable not using but for - Substantial factor test If fire caused by d's neg actions can be shown to be a substantial factor of p's injuries - which also involved neg action of an unknown source then d will be held liable for entire damages Why isn't d only partially liable? His neg action alone still would have caused the same amt of damage Double fault and alternative liability -Problems in determining which party caused the harm Up to d to show that other d caused harm CASE: Summers v. Tice (SC Cal, 1948) Pg 285 Quail hunting p summers and two d's (tice and simonson) go quail hunting. Both ds shoot at quail and one of the shots strikes p Both ds can be held liable for negligently shooting p when there is no evidence of who actually shot p. The burden in this circumstance is on each of the ds to prove the shot did not come from their gun but the other ds gun -compare to Ybarra v. Spangard(multiple hospital employees held liable without knowledge of exactly which one caused harm) Both d's actions were negligent (shoot at quail but hit a person - isn't this an intentional tort) 50/50 chance that it was either - this does not satisfy the probability requirement for causation link
48

What injustice occurs if we hold both d's liable - 1 d held responsible for an injury he did not cause Ybarra - people who did nothing will be held resp unless they speak up and say who did But the summer v tice d's not acting in concert - as hospital workers were - Acted negligently but separately - concurrent tortfeasors This variation on but for puts burden of proof in hands of defendant - d in better position to show evidence they were not the cause (weisberg not sure of this) Market Share theory Double fault and alternative liability extended to 3+ defendants /especially with products liability cases - each has burden of proof CASE: Sindell v. Abbott Lab (SC of Cal, 1980) Pg 287 Estrogen Pill approx 195 manufacturers used identical formula to produce estrogen pill - P alleges her mother took a synthetic estrogen pill during pregnancy resulting in p's developing cancer. P unable to show which manufacturer made the pills her mother took. She sues 5 manufacturers who control 90% of syn est pill market p does not need to id one manufacturer of drug her mother used in order to establish a cause of action for negligence in order to hold group was negligent unavailability of proof not p's fault - d's have to assume the burden - from a policy stand point also d's better equipped to bear burden . Each d free to prove they did not manufacturer drug taken by p's mom - otherwise they will have to share in any recovery for damages In Sindell - DES link to breast cancer was proven - but problem with d's sued - only part of manufacturer sued (5/195) but they made up 90% of manufacturers - and were more likely than not to have manufactured the pills the p's mother actually took Shifts the burden to these d's - they have burden of proving they did not manufacture pills taken-all d's not included in this case Already known that des caused damages Joint and several liability Joint liable - each liable up to full amount Several - parties responsible for only their respective obligations Sindell - each company held individually liable and have burden of proof to get out of the suit

B.PROXIMATE CAUSE Proximate cause - any cause which in natural and continuous sequence, unbroken by an efficient intervening cause, produces the results complained of and without which the result would not have occurred - Pg 321 (Yun v. Ford) JOINT TORTFEASORS Liability and Joinder of Defendants CASE: Bierczynski v. Rogers (SC of Del, 1968) BIKE RACE d1(race) and d2 (biercznyski) racing on highway when d1 struck p's car d2 was not directly involved in accident. d who was engaged in negligent act of auto racing can be held liable for damages to p when his vehicle was not direct cause of p's injuries and damages delaware has no statute on auto racing on highways - Not negligence per se but still negligent
49

Generally held that parties engaged in auto racing are equally held liable for any harm caused to a third party regardless of which parties vehicle caused the damages All parties are wrongdoers acting in concert and are therefore each is liable -Each has induced and encouraged the tort Joint and several liability - each of several tortfeasors is liable jointly with others for the amount of the judgment against them, and that each is also individually liable for full amount. P can collect from any one of them or any group(subgroup) . For joint tortfeasors: d's had to have acted in concert and be in exclusive control, and all be before the court Proximate Cause - Temporal Remoteness - Public Policy CASE: Enright v. Eli Lilly & Co. (NY COA, 1991) Patricia Enright's mother takes the drug DES manufactured by d while pregnant in 1960 Enright born in 1960 - in her adulthood suffers several miscarriages and has a premature child - Karen Enright (P) who has cerebral palsy. P sues drug company Karen Enright cannot recover for damages caused by d's drug being used by her grandmother Public policy holding drug company liable for third generation effects of a drug may deter some beneficial drugs form entering the market Cause-in-fact established but p is tracing negligent act back to her grandmother - NY court decides not to take the extra step Albala precedent - pre-conception tort against mother was held non-actionable Burden is too huge to implicate the d for the resulting brain damage of child CASE: Atlantic Coast Line R. Co. v. Daniels (COA of Georg, 1911) Pg 293 ISSUE: where does causation begin and end? REASONING: Causation link has to be limited or else reductio ad absurdum - reduction to the absurd can occur causation has to end or begin somewhere - d can claim that if p had never been born the injury would not have occurred Have to look at the 'nearest' cause - Can apply to a time limit as well (Enright v. Lilly) If cause in fact not present no need to look for proximate cause Cause in fact refers to cause and effect relationship established with but for or substantial factor test If we are looking for proximate cause then cause in fact has been established -have decide if the cause in fact creates a legal liability - usually determined through policy concerns use of logic, common sense, policy, precedent, and idea of justice If proximate cause doesn't establish legal liability then regardless of cause in fact there is no liability UNFORESEEABLE CONSEQUENCES - Remoteness CASE: Ryan v. New York Central RR Co (NY COA, 1866) Pg 294 D's woodshed caught on fire through negligence. Fire spread to p's house 130ft away and house was destroyed. House was in a populous area and other houses were also burnt. Person held liable for the consequences of their own acts and liable for damages with proximate results of those acts but not for remote results. The line is hard to determine. Compared to engineer who negligently lets steamship coal fire spark burn another's house - prox cause is satisfied - But if fire then spreads to another and another and another house - is engineer liable for all that damage?
50

The original house - yes because the result was direct, natural and expected as a result of the negligent act, The other houses - other factors entered into the result Yes, fires are known to spread but to what extent they spread depends on other factors such as wind direction and condition and materials of adjoining structures etc. - these the negligent party has no control over Damages incurred not immediate but remote result of negligent act therefore d cannot be expected to be liable - Policy concerns of attributing unlimited liability Insurance comes in to play - court basically tells p to get his own insurance **modified to first adjoining landowner and not the first building Examples we have seen of spatial remoteness - too far away physically: Palsgraf - was p just too far from the official act of railroad Ryan v. New York RR- 130 feet from original fire of woodshed negligently caused by d. Compared to Vaughan v. Menlove - hayrick case - d in this case wanted the individual subjective standard - looking at duty standard. Pg 145 Temporal remoteness - too far away in time: Enright v. Eli Lilly - p is granddaughter of person who took the estrogen pill. Eggshell Plaintiff CASE: Bartolone v. Jeckovich (SC of NY, 1984) Pg 297 P involved in 4 car chain reaction which d's were found liable for. P claimed to have suffered psychotic breakdown as a result of the crash - the injuries sustained aggravated a pre-existing schizo condition. P's quality of life drastically reduced as testified by 3psychs and 1 neuro. Other bad things happened to him as well. p's mental breakdown can be attributed proximately to d's negligent actions when breakdown was due to aggravation of pre-existing condition. Use Bonner v. US, Steinhauser Defendant must take a p as he finds him - can be held liable for damages for aggravating a pre-existing condition even if the injury is unforeseeable Compare with Vosburg v. Putney - Defendant must take p as he finds him Damage was not remote even if it was unforeseeable Includes the concepts of 'remoteness' and 'the bizarre' Unforeseeability of consequence of negligent act - aggravate a pre-existing mental condition Whiplash was a foreseeable consequence - but the extent of it is not used to determine liability 3 kinds of unforeseeable situations 1-Unforeseeability of extent of the damages/injury - generally will not relieve d of liability -Bartolone - Knew I was causing whiplash but not the mental breakdown -Vosburg - Knew I was invading person but did not know you would lose your whole leg Direct Causation CASE: In Re Arbitration Between Polemis and Furness, Withy & Co., Ltd (COA, 1921) Pg 300 Respondents contracted their vessel to appellants to carry cargo of benzine and petrol to casablanca. While unloading at casablanca - plank fell into vessel where petrol was stored and caused an explosion - Respondents suing for loss of vessel due to negligence of apps employees - appellants claim damages were too remote
51

Arbitration court award 196pounds - ship lost to fire - based on foreseeability of some damage from plank falling. fire started from spark igniting petrol, spark caused by falling board striking something in the hold Appellant was responsible for all of the damage caused by negligence even when that damage was a remote and unanticipated result. I - falling board due to negligence of employees Fire direct result of board falling - immaterial that spark was not reasonably anticipated Can't say appellant Could not have reasonably foreseen the extent of the damage Anticipation of a type of damage is immaterial 2 - type of damage not material as long as it can reasonably be traced back to negligent act directly. Consequences which follow in unbroken sequence, w/o an intervening efficient cause , from the original negligent act are natural and proximate Contrast unforeseeablility of result and unforeseeability of manner in which result is brought about. -generally the latter will not relieve d of responsibility For intentional torts very remote causation can be used Anticipation(reasonable risk) of a result - early causal connection Neither knowledge nor duty to foresee - direct and immediate results considered only Accidental to negligent dropping of plank - the p proves this Not covered by the rule of the trombone case Different kind of unforeseeability - the fire was not foreseeable but the plank falling was - and the foreseeability of some damage happening was foreseeable 2 - Unforeseeability of type/kind of damages - this may be able to relieve d of liability No foreseeability of the particular type /kind of damage - Blythe v. Birmingham But with direct causation -Polemis - Risk of injury by impact v. risk of injury by fire or explosion - type of damage unforeseen! - absence of foreseeability/not of some damages arising but of type of damage that would occur (Direct line of causation - type of injury immaterial - Polemis) Unforeseeability of type/kind of damage IS material to proximate cause CASE: Overseas Tankship (UK) Ltd. v. Morts Dock (Privy Council, 1961) Pg 302 Molten metal - Wagon Mound 1 P - Morts Dock had ship building and repair wharf. Overseas (D) owned a freighter moored at morts dock which spilled oil in to the harbor. The oil caused minor damage to dock and p's brought no action against that. The oil eventually caught on fire and destroyed the wharf and p brought suit A negligent act that results in a foreseeable damage should not also hold the actor liable for all other unforeseeable consequences, even if direct. Foreseeability important to establishing a legal cause would the reasonable person foresee. It is not the act but the consequences on which tortious liability is founded. Action brought by a for damage negligently caused by B - a fire due to careless spillage of oil. Only liability in question is the liability for damage by fire. The foreseeability of damage to one person should not affect the unforeseeability of damage to another party arising out of the same negligent act The test of liability for shock is the foreseeability of injury by shock - substitute fire Prof Keeton - a negligent actor is legally responsible for that harm and only that harm of which the negligent aspect of his conduct is a cause in fact. p's were contributorily negligent in wagon mound 1 and could not raise a foreseeability of risky act producing a result argument so argued: The test for liability for shock is the foreseeability of injury by shock - (foreseeability of consequences)
52

Foreseeability of Type/Kind The area within which liability is imposed is that which is within the circle of reasonable foreseeability Case: Overseas Tankship (UK) v. Miller Steamship (Privy Council, 1966) Pg 306 Wagon Mound 2 Wagon mound 1 facts - this suit brought by two ship owners whose ships were destroyed in the wharf fire on nuisance and negligence grounds D can be held liable for damage caused in fact by his negligent actions even if consequences of negligent act were reasonably foreseeable by the ship's engineer although the risk was slight. A reasonable man would neglect even a very small foreseeable risk only if there was a valid reason for doing so i.e. cost to avoid risk too great Freighter should have tried to avoid the risk - had a duty to stop the discharge of oil into harbor While it may not have been a probable consequence in the reasonable person's eyes it was a possible consequence and if the gravity of injury proved to be greater than the cost of avoiding the breach of duty the actor should do so. Victim in the Unforeseeable Zone of Danger CASE: Palsgraf v. Long Island RR (COA of NY, 1928) Pg 308 Exclusion of the Unforeseeable Victim Plaintiff was standing on a platform of RR. A man was trying to get on a train, and he ran to get on. A guard held the door open for him, and the man looked like he was about to fall. Another guard pushed him from behind to help him not fall, and the guard on the train reached to help him. In the process of trying to help, the package that the man was holding fell and it was a package of fireworks and they exploded. It was not evident that they were fireworks from the outward appearance of the package. The explosion made some scales at the other end of the platform fall down and then they hit the plaintiff and injured her. P sues. Issue: Does a defendant owe a duty of care only to those plaintiffs who are in the reasonably foreseeable zone of danger? Holding: Yes. Reasoning: 1. The conduct of the RR employee was not negligent at all in respect to Palsgraf. 2. No one knew that there were fireworks in the package. 3. Before negligence can be determined, it must be determined whether there was a duty of care to the plaintiff and if the injury could have been avoided if the duty had been observed. 4. The package was packed in such a way that no one would really know what was in thereso even if the guard had thrown down the package purposely, as far as the package appeared, he would not have threatened the plaintiffs safety 5. So, liability can be no greater where the act is inadvertent. DISSENT: Justice Andrews thinks that the defendants duty of due care is owed to anyone in the world who suffers injuries as a proximate result of the defendants breach of duty. D is liable regardless of whether injury was foreseeable or not as long as the injuries were proximately caused by the defendants negligence toward someone. Polemis and Andrew's Dissent: Foreseeable if you can Directly trace injury back to negligent act - Type of damage is irrelevant Polemis is talking about foreseeability of unreasonable act of risk creation - a breach of duty that is backwards looking Wagon Mound 1 and Cardozo Palsgraf Only foreseeable consequences
53

Duty and the burden determined by whether defendant's conduct involves an unreasonable risk of harm ro plaintiff - the damage to her was not foreseeable therefore the fact that the conduct was unjustifiably risky to someone else is irrelevant. Palsgraf - negligence is a tort if there is damage The risk reasonably to be perceived is the duty to be obeyed D<FG Notion of risk creation Defenders of Foreseeability --> more risk prone than direct causation people Direct causation - as soon as you are acting negligently you are negligent - Cardozo says negligence on the air will not do Andrews is more risk averse and moralistic- higher standard -ideal Cardozo argues that for regular people life could not go on if held to this high standard Encourages people to be less risk prone - huge burden - upsets carroll towing analysis Another subtheme of palsgraf majority opinion - actions that start out intentionally - direct causation would probably be longer Intentional tortfeasor treated more morallistically - punitive damages Foreseeability looks forward to consequences of the negligent act (second foreseeability chance) And direct causation looks back - at original negligent act Case: Pg 318 n.9 Kinsman - always policy stopping points that even direct causation must use - jury determine if injury could not have happened without the negligent act - substantial factor - look at intervening factors - if a superseding cause can end liability of original negligent actor 1 -No limitation of damages to consequences foreseeable at the time of the negligent conduct when the consequences are direct, although other and greater than expectable, is of the same general sort that was risked 2 - wheat company lost wheat supply on a ship stuck from the whole mess - too remote or indirect a consequence of the d's negligence NOTE for damage to property there is tort recovery but for economic loss there is no tort recovery Pg 305 n a - modification instead of 3rd person what if it had been original p who was served shrimp? 3 - Unforeseeability of the manner in which the damages/injuries are brought about- generally will not relieve d of liability n.3 Pg 301 (Bunting v. Hogsett) Extraordinary Intervening Cause --> Superseding Cause CASE: Yun v. Ford Motor Co. (NJ Sup Ct, 1994) The spare tire fell off of Yuns van and rolled to the left side of the highway. Chang got out of Yuns car and ran across the highway to get the spare tire. While running back, he got hit by a car and died. Yun sued a whole bunch of people including Ford the driver of the ford car-Lindermann, Universal installed spare tire assembly, etc. All of them got summary judgment except for the driver and owner of the car that hit Chang. Yun appealed the judgment - issue of proximate cause is one for the jury to decide. Not appealing against ford - spare tire not part of original car Kim - mechanic - dropped out - no legal basis - had warned the p Products Liability Act Have to prove causation still -the defect of the spare tire bracket assembly proximately caused Chang's injuries
54

The defendant shall not be found liable when a superseding cause other than the defendants negligence is responsible for the harm. The court basically says that Chang was really stupid and shouldnt have gone running across the street. The court says that this unforeseeable conduct basically breaks the chain of proximate causation that might otherwise hold at least some of the defendants liable. Looking at whether chang's conduct was reasonably foreseeable v. Highly extraordinary The dissent says that the case is not such a slam dunk that it should be taken away from a jury. The minority says it is debatable whether the defective spare tire assembly caused the plaintiffs death. The judge makes the point that judges in general are more conservative than the public at large, and what seems unreasonable and imprudent to a judge might seem like reasonable behavior to the average person. Conclusion: The dismissal of the suit was upheld, but the New Jersey Supreme Court later overturned the ruling of this court and reinstated the suit. -found against the p using direct causation, broken by extraordinary/superseding act Foreseeable Intervening Causes --> Not Superseding CASE: Derdiarian v. Felix Contracting Corp. (NY COA, 1980) P Derdiarian employee of subcontractor hired by D Felix - was sealing off a gas main D Dickens had a seizure lost control of his car and struck p throwing him into the air. P was also splashed with hot liquid enamel also thrown into the air - ignited into a fire ball D felix added for negligent safety precautions. Called expert in traffic safety as a witness - who spoke of lack of a barrier around work site. Felix argues it was a freak accident - intervening cause was dickens - no causal link D can be found liable for injuries sustained by p when the cause in fact was negligent safety precautions but the intervening cause was a negligent driver. D can be found liable - jury must determine the foreseeability of an intervening cause Causal link not always automatically severed by an intervening cause - foreseeable intervening causes will not break the chain - only extraordinary intervening causes. Driver held liable? Probably not - but his liability does not insulate felix's Negligence of third party sometimes not unpredictable Superseding cause - not always going to depend on the nature of the intervening act Where the risk of the intervening act occurring is the very same risk that rendered the defendant negligent then not a bar to d's liability If intervening act not criminal, extraordinary, intentional and possibly not even negligent - then not superseding Pg 328 n. 10 - p's intervening act - contributory negligence - has to be more than contributory negligence to be superseding Intervening Causes - Intentional Acts v. Negligent Acts CASE: Watson v. Kentucky & Indiana Bridge & RR Co. (KY COA, 1910)Pg 329 D's gasoline tank spilled oil into the streets. Duerr a pedestrian dropped a lit match and started the explosion. Duerr says he did not know, a witness says he did it maliciously. P was injured by explosion and sues D D is not necessarily liable if intervening cause was intentional and unexpected. Jury needs to decide if the Duerr - the intervening cause - acted intentionally or negligently and that can then to determine whether the intervening cause was sufficient to cut off the liability of the d.
55

Duerr case - foreseeable that gas might be set on fire - but was the manner in which it happened (with an intervening cause) an important factor even if the outcome could be foreseen -in kentucky - criminal conduct no longer interrupts the chain Intervening Cause of Suicide - irresistible impulse can be superseding cause CASE: Fuller v. Preis (NY COA, 1974) Pg 335 p executor of decedent's estate suing d who struck decedent. Decedent sustained head injuries from accident that created seizures. Decedent eventually committed suicide. Witnesses testified that he had an irresistible impulse created by the head injuries. Decision: jury verdict of $200,00 for p. app div jnov for d and dismissed complaint. P appealed. Coa reverses app div and orders new trial (because of dismissal of complaint) An act of suicide is not a superseding cause in negligence law that cuts off liability. But if the result is foreseeable then the initial tortfeasor may be held liable. Jury should be allowed to decide if the intervening cause of the doctor committing suicide was intentional act without the intent actually being a proximate cause of the injuries sustained - Was the suicide an irresistible impulse enough to void intention or criminality For irresistible impulse to be proven - has to be done in a frenzy w/o signs of controlled or planned action Temporal remoteness - 7 mos - may or may nnot break chain of causation For irresistible impulse to be proven - has to be done in a frenzy w/o signs of controlled or planned action Question of the agency in this case, unlike the agency of duerr or the thief in Ney (pg 212) Rescue Doctrine CASE: McCoy v. American Suzuki Motor Co. (WA SC, 1998) Pg 338 P McCoy witnessed a suzuki drive off the highway and roll, he helped the driver and the police holding out lights to direct traffic for 2 hrs. Police left him there after the scene was cleared and McCoy was then hit by another vehicle. McCoy sued the 3rd driver, the suzuki driver and passenger, and suzuk and the state for police's negligence (should have waited for incident to have quieted before leaving) Rescue Doctrine does apply to prod liability cases and proof of proximate cause is necessary Rescue Doctrine: 1 - d was negligent to person rescued and caused negligent danger to rescuer 2-danger was imminent 3-reasonably prudent person would have thought there was danger 4-rescuer acted with reasonable care P fulfills all these and can use rescue doctrine The rescuer still has to show he was proximately injured by d's original negligent act. (Maltman helicopter crashed on way to rescue - no proximate cause) Foreseeability of a rescuer being injured while rescuing someone from a suzuki car has to be decided by a jury Applying to prod liability? Yes, rescuers in relation to defective products like cars can be reasonably anticipated Incident may have been quieted but relies on the original actor foreseeing that a party might come to the harmed party's aid and in turn be harmed also - danger invites harm Duty owed to rescuer? The negligent defendant could have a duty towards someone they didn't foresee who reacts to the injuries of their negligent act
56

Can rescue doctrine be countered by assumption of risk argument? P is absolved from such an argument but not professionals - Pg 342 n.8professionals not able to recover under the rescue doctrine because already compensated through their jobs (firefighter rules) Public policy - no duty to rescue but don't discourage if decide to help Public Policy - Social Host Liability CASE: Kelly v. Gwinnell (NY SC, 1984) Pg 344 D spent an hour or two drinking at co-ds home. Afterward, Co-ds accompanied him outside, chatted, and watched him as he drove off to go home. En route d was involved and caused a head on collision with p who was seriously injured. A social host who enables an adult guest at his home to become drunk is liable to the victim of an automobile accident caused by the drunken driving of the guest The reasonable and prudent person at the time and place should recognize and foresee an unreasonable risk or likelihood of harm or danger to others. When negligent conduct creates such a risk, setting off foreseeable consequences that lead to injury, the conduct is deemed the proximate cause of the injury. A reasonable person in ds position could foresee quite clearly that this continued provision of alcohol was making it more likely that co-d would not be able to operate his car carefully. Thousands of people are killed every year as a result of this activity. The Gov. imposes a criminal sanction for violations of duty to refrain from this activity in accordance with the states policy. A host who serves liquor to an adult social guest, knowing both that the guest is intoxicated and will operate a vehicle, is liable for injuries inflicted upon a third party as a result. Plaintiffs Argument: D provided his guest liquor in his home, knowing that thereafter the guest would have to drive in order to get home. Defendants Argument: Ds level of intoxication was unknown by d & co-d when he left the house. Cod had no control over ds actions, and could not use force to keep d from leaving. What are the arguments in favor of social host liability? It forces social hosts to be responsible when theyre serving guests in their own homes. It may prevent drunk driving. We want social hosts to recognize when guests are intoxicated and shouldnt be given any additional alcohol. We want to make sure that victims are compensated. A drunk driver might not have enough money to compensate the people they injure. With social host liability, you have another pocket to pick to get money for the victim. We want to deter drunk driving. Also, social hosts are already liable for serving alcohol to minors. Licensees are also liable under the Dram Shop Act. The court suggests that they have surrounded the social gathering situation with liability, and so its not such a big step to, in effect, fill in the gap. What are reasons not to have social host liability? Well, how can a social host know what a visibly intoxicated person looks like? It puts a burden on social hosts that they may not be in a position to bear. On the other hand, a bartender has enough experience to know when to stop giving people drinks. On the other hand, theres a possibility that you could serve someone who is drunk more alcohol and become 100% liable for the harm they cause (under joint and several liability) if the drunk person doesnt have any money. Some would say that fault should lie with the drunk driver. Also, sometimes a social host is drinking, unlike a bartender. A social host also will have difficulty in preventing a guest from departing. Commercial establishments are more likely to have insurance to cover liability than private people. In that way, the businesses can spread the costs of potential liability to their customers by raising prices. We also dont want to inhibit social gatherings! The
57

dissent says that those are important! We say that there is a public policy in favor of gathering together. Its hard to know when youre a social host what lengths you should go to in order to stop a drunken guest. Synthesis CASE: Marshall v. Nugent (1st Cir, 1955) Pg 356 Truck driving at excessive speed - assess total risk merely from that action But if foreseeability of intervening forces are taken into account the risk may be multiplied CASE: Diehl v. Fidelity Trust Co. pg 360 Metropolitan negligently discharged steam that created ice on sidewalk in front of fidelity building fidelity had notice of icy sidewalk and negligently did not clean sidewalk - p slipped and was injured on ice. Both met and fid are equally held liable even if fid had notice and did not fulfill their duty. Fid liable because original tortfeasor - they become joint tortfeasors But parent who intervenes to take dynamite caps away from child but then leaves them where child can get to them anyway takes the liability away from construction company. Parent relieved the construction company of any responsibility 4-DAMAGES Case: Dillon v. Twin State Gas & Electric Co (New Hamp SC, 132) Pg 400 P dillon executor of his son's estate against d - twin state p's son 14 years of age played on d's bridge, along with other boys Bridge had wires that were often not electrically charged - insulated only for weather and not electricity. p's son lost his balance and grabbed onto a wire that was charged and was electrocuted Issue: how are damages to be calculated in view of the negligence of the d tempered by the inevitable injury of some level of the p due to his own negligence? Holding: jury has to decided the question of negligence in terms of how much damage was actually done by the wires themselves and not by the fall Reasoning: D has a duty to the p - since wires were electrocuted But p's son was already in the midst of falling and without the wires could have been still seriously injured or killed Value of p's sons life has to be measured by his anticipated future earnings - in this case tempered by the minimal amount of damage that would have occurred if wires had not been electrified but p's son had just fell and was injured nonetheless Damages can be reduced in situations where someone already has some existing condition. If you kill someone with cancer, theyre already likely to die, and so their damages are likely to be reduced. The value of someones life is to be measured by their anticipated future earnings and other factors. 4. NEGLIGENCE DEFENSES Plaintiff's Conduct (A) Contributory Negligence Doctrine - p who is negligent and whose negligence is a proximate cause of his or her injuries is totally barred from recovery (common law) CASE: Butterfield v. Forrester (King's Bench, 1809) Pg 586 FACTS: P was riding along road where d had negligently placed a pole while doing work on his house, and crashed into pole sustaining injuries. Pole only blocked part of the road. The p left a pub at 8pm
58

with enough light to see 100 yards ahead. Witness stated p was riding fast and would have avoided pole if he weren't. p was not intoxicated Judge directed jury to decide if a person riding with reasonable and ordinary care would have seen and avoided the pole and if they decide p was riding at a negligent speed to find for d A p can be totally barred from recovering for damages due to d's negligently obstructing road when the p was riding without reasonable and ordinary care. One person being at fault will not dispense with another's using ordinary care for himself Deters p's from being negligent notes: How to treat p's negligence: 1-completely bar claim (Butterfield) 2-workers comp/no-fault auto accident 3-exceptions to the first 2 methods 4-compare p and d's fault levels - reduce p's recovery based on proportion of p's fault (comparative negligence) Reasons for contributory negligence Pg 588: 1-Penal basis - punishment 2-P has unclean hands 3-Encourages optimal care by both parties - incentive to use ordinary care - what does optimal care mean? Reciprocal risk creators - how does contrib maximize reasonable care? Have to anticipate someone else's negligence 4-p's contrib cuts of proximate cause of d's negligent act (superseding cause) Burden of proof is on the d to plead contributory negligence Jury often decides on contrib p's contrib has to be a cause in fact (has to follow negligence formula) - a substantial factor Proximate Cause of contrib also has to be assessed D's Intentional conduct - contrib is often NOT a defense to an intentional tort Differs from negligence usually in degree of risk and kind of risk P's Rebuttal - Last Clear Chance Doctrine- limit on the contributory negligence defense - if just before the accident the d had an opportunity to prevent the harm (and the p did not) then the existence of the opportunity clears the defense of the p's contributory negligence. d's failure to avoid the risk could be seen as a superseding cause - attempt to lessen harshness/concreteness of contributory negligence CASE: Davies v. Mann (Exchequer, 1842) Pg 591 P tied his donkey's feet together and left it very close to the road. D negligently rode fast down the road and ran over donkey with his horses killing it. Judge instructed jury to decide whether the d proximately caused death of p's horse even though p negligently tied up donkey in road when the d may have been able to avoid the accident if he had used ordinary care. he had a chance of avoiding accident by using reasonable care - riding at a slower pace - otherwise the p would have been barred completely due to his own contributory negligence of leaving horse in the road - instead the whole loss is placed on the d notes: Last clear chance can be applied depends if 1-p is helpless to avoid danger or 2- merely inattentive
59

Ameliorate contributory negligence: Even at contributory negligence - jury would still decide the amount of damages based on negligence even though it would be said that p was at fault and no recovery - jury would then say p not culpable of contributory - but 95,000 would be awarded ($100,000 - 5% NEGLIGENCE OF P) Why the shift to comparative neg? Now jury can explicitly negotiate damages based on levels of negligence of both parties (B) Comparative Negligence - rejects the all-or-nothing approach of contributory negligence - and instead divides liabilty between p and d with regards to their relative degrees of fault - p's recovery will be reduced by a proportion equal to the ratio between his or her negligence and the total negligence contributing to the accident. 1. Pure comparative negligent state-the P can achieve a recovery even though you were more negligent than the other party **this is the majority** 2. Partial comparative negligent state-cannot get a recovery if P more negligent than the other party (49% and 50%) CASE: McIntyre v. Balentine (SC of Tenn, 1992) Pg 592 P Mcintyre sustained injuries in car crash with D-Balentine. P merged south from truck stop onto highway at same time d was driving south on highway. Both p and d were intoxicated, and d was speeding. P brought suit, d claimed defense of contrib . Jury found contrib and completely barred p from recovery - verdict for d. ISSUE: whether p and d should equally be held liable for negligence when both are equally at fault. HOLDING: yes, comparative negligence should be allowed REASONING: All or nothing bar too harsh But there are exceptions to ameliorate contributory negligence: -d's conduct intentional -D was 'grossly' negligent -D had a last clear chance which through use of ordinary care would have avoided p's injuries -p's negligence remote Fed rule does use comp negligence for certain p's And other states also have setup exceptions Two forms of comparative negligence: 1-Pure - p's recovery reduced in proportion to the percentage of negligence attributed to him 2-Modified - p's recover according to pure formula but only if it either - 1-does not exceed 50%or 2-is 49% or less than the d's percentage of the negligence Tenn court adopts the modified comparative 49% or less negligence -This does away with last clear chance and remoteness and instead examines degrees of fault for each party -Does away with joint and several liability in some jurisdictions(?) **NY adopted the pure form of comp negligence D can allege affirmative defensive that a non-party contributed to the negligence - joinder (C) Assumption of Risk - p is said to have assumed the risk of certain harm if he or she has voluntarily consented to take chance that the harm will occur. When the assumption is shown, p (at common law) is completely barred from recovery. (1) Express - express words that show knowledge and assumption of risk - scope of the risk has to be what is consented to CASE: Seigneur v. National Fitness Int (COSpecialA, MD 2000)
60

Exculpatory clause in a fitness contract - Provision in a contract under which (1) one party is relieved of any blame or liability arising from the other party's wrongdoing, or (2) one party (usually the one which drafted the agreement) is freed of all liability arising out of performance of that contract. P - Seigneur signed contract with D - Nat Fit - disclosed she had back problems and poor fitness upon signing - contract included clause relieving nat fit of liability During fitness evaluation p tore shoulder, filed suit - nfi motioned for dismissal based on clause P said clause not valid - public policy reasons - and contract unclear D cannot be held liable for injuries to p when p has signed a contract with a valid exculpatory clause. d is not liable for injuries - exculpatory clause is sufficient to insulate the party from his or her negligence as long as the clause clearly indicates d's freedom from liability for personal injury caused by the d's negligence. REASONING: Validity - contract is valid: -Party protected by the clause does not intentionally cause harm or engage in acts of reckless, wanton, or gross negligence. -Public Interest - Not an essential service - does the clause violate public policy - inn, hospital - essential services should not have exculpatory clause -Bargaining power was equal -Scope - contract has to pertain to situation - injury has to be within the scope of risks that the exculpatory agreement covers (2) Implied - p indicates by their conduct that they know risk is in question and agrees to bear the risk bars p's recovery 2 part test: Knew of the risk and Voluntarily assumed the risk b. Exceptions to Implied Assumption of Risk: i. No other viable alternative1. Rush v Commercial Realty Co- P fell through a trapdoor in an outhouse for tenants. (and imbalance of power) ii. Acted in emergency, not held to have assumed the risk. 1. Can be your own emergency or someone else's. 2. Ex: Speeding car-saves kid, hurts himself-it was an emergency so voluntarily assuming the risk is not held against him. CASE: Rush v. Commercial Realty (SC of NJ, 1929) Pg 607 P's tenants of d's house. They fell through a trapdoor in the only reasonably available outhouse The p did not assume the risk of falling through a trap door when she entered a negligently maintained outhouse Duty of care to tenants to properly maintain the outhouse - Only a question of whether p was contributorily negligent - she was not - she had no knowledge of trap door and no choice but to use the only available bathroom - and jury decided as is correct Volenti non fit injuria - to the willing, there is no injury Person has to know of the particular risk/ appreciate the magnitude of the risk/ and voluntarily encounter the risk E.G. : Driving instructor assumed the risk of driving with a new student - voluntarily took on the risk example of d's negligence being suspended - no duty on the part of the student
61

Contributory Negligence v. Implied Assumption of Risk Comparative negligence does not remove assumption of risk as a defense if it is express But When P is acting carelessly, then comparative negligence takes over so dont need assumption of risk. CASE: Blackburn v. Dorta (SC of Flor, 1977) Pg 610 ISSUE: Does implied assumption of risk serve separate function from contributory negligence? HOLDING: the affirmative defense of implied assumption of the risk is merged into the defense if contributory negligence and the principles of comparative negligence shall apply in all cases where such defense is asserted. REASONING: Express assumption of risk - not discussed in this case Implied assumption of risk has two categories: Primary - d owed no duty or did not breach duty owed - (ex. sports injuries) - d reasonably believes that the p understands the risk and is voluntarily submitting to it - complete to bar to recovery even in comparative juris Secondary - breach of duty occurred but d has an affirmative defense Pure/strict - reasonable but bars recovery by percentage of negligence in comparative juris (rush into burning building to save infant) Implied Qualified - unreasonable and voluntarily - bars recovery (rush into burning building to save hat) - could be considered contributory negligence - and therefore flows to comparative negligence standards analysis Negligence Defense can be used for: Strict Liability - assumption of the risk - contributory negligence D violates Statute - negligence per se - contrib can be used as a defense - some statutes abrogate the use of the defense Statutes for p's unable to protect themselves - child labor, statutory rape etc. - contrib and assumption of the risk can't be used as a defense P has to show d was negligent and his own absence of negligence 5. VICARIOUS LIABILITY 1) One person is being held liable for the tortious activity of another. 2) Respondeat Superior-look to the person higher up. a. an employer will be liable where it was within the scope of employment. b. Test for Scope: If employment context creates the necessity that then turns into the accident, then the employer is liable. c. General duty: driving to and from work is not in the scope. There are 2 main rationales for vicarious liability: o The control theory-liability where the employee acts w/ the acquiescence of the employer o The enterprise theory-whenever the employer would have benefited by the context of the act of the employee but for the unfortunate injury. Spread to the deep pockets. An employer can be held liable for own negligence in hiring an incompetent employee. Independent Contractors: a. Generally employers not liable for their tortious acts b. Exceptions: i. Contractor is engaged in an inherently dangerous activity ex: blasting
62

ii. For public policy, the duty is non-delegable. Ex: motorist, business c. Where there is an independent contractor who acts beyond the supervision, dominion, and control of the employer, the employer will not be held vicariously liable for injuries caused by the independent contractor. If an employee commits a tort during the scope of his employment his employer will jointly be liable (let the person higher up answer) -cost of doing business -employer has deep pockets and/or insurance and employee may be judgment proof Employee is subject to control of person who hired him - not independent contractor Employer has control over the physical details of the work with an employee CASE: Murrell v. Goertz (OK COA, 1979) D Goertz worked paper published by co-d ok pub co, while collecting delivery dues from p, p questioned d about damage to her screen door from newspaper being thrown at it, p and goertz got into an argument where upon p slapped goertz and hit her back, p sustained injuries and need medical attention. P sought $52,500 for damages, med bill, pain and suffering and exemplary damages., p also sued co-d as goertz's employer ok pub co cannot be held liable for damages to p caused by one of their employees who was an independent contractor and who they did not have control of physical detail of the work over? Goertz was hired by an independent contract working for co-d, ok had no contactwith goertz at all and could not be considered a servant of defs Typically independent contracts are seen to be in control of their own risk Foster parents are independent contractors and not direct employees of the state - Although there are some exceptions 6. STRICT LIABILITY -liability w/o a showing of fault PRIMA FACIE CASE: a. The existence of a duty on the D to make it safe b. Breach of that duty c. Breach was actual and proximate cause of the injury d. Damage to the Ps person or property. Strict liability goes back all the way to anonymous where were liable for your actions without requirement to show fault or intentIn a way we are back where we started. May recover for personal and property damage, but not economic. Strict Liability - a distinctly different regime from the torts "baseline" which is negligence A re-focus to the activity (or thing) itself; and not the way it is conducted Strategically if negligence can be demonstrated it is second best to use strict liability Negligence means fault - don't have to have fault to prove strict liability Fault is something jury can get its teeth into morally A.Animals Strict liability imposed on those who keep, possess, or harbor the animal, not just the owner 1-Trespassing animals - owner of the kind of animals that are likely to roam and do damage is strictly liable for their trespasses (or intrusion since no intent) Exception for household pets like cats and dogs And for animals in transport for lands adjacent to the road Fencing out statutes - strict liability when d's animal breaks through
63

Fencing In statutes - strict liability when d's animal breaks out No liability without fault (p has to show d's intent or negligence) County statutes in some areas with one of the four 2-Wild Animals (ferae naturae) - strict liability if animal injures someone Customs of a community influence the classification of a wild animal 3-Domestic Animals - dog or cat entitled to one bite at common law But more commonly - For domestic animals strict liability if the owner knew or had reason to know the animal had vicious tendencies Strict liability - d must pay damages even though the d did not fail to live up to the objective standard of reasonable care Still have to establish causation and damages and there are defenses to strict liability B.Abnormally Dangerous Activities Usually strict liability exists for abnormally dangerous activities / ultrahazardous CASE: Rylands v. Fletcher (Eng, 1866) Pg 692 P's coal mines were adjacent to land where d was building a reservoir. When d partially filled the reservoir with water the water broke through some of the mine shafts and flooded p's property. d's had no knowledge of the abandoned mine shafts- no negligence or fault. Lower court verdict for d. Int appeals ct verdict for p. exchequer held for p. P cannot recover for damages neither negligently or intentionally created by d's actions. REASONING: Lower court said there was liability because person who has dangerous things on his property is obligated to maintain care and control of them, if such dangerous things should escape his control and injure someone off of his property then he should be held liable - like dangerous animals escaping Higher court - liability exists because what d had on their land was unnatural to that area (if it had been natural no liability) Escape from liability by showing that the escape was p's fault or an act of god In this case Independent contractors were the ones at fault - miller - fault does not transition to employer (respondeat superior not available) -only need to know that if the negligent actor (usually an employee) is an independent contractor then the d shall not have that liability imputed to it After no negligence available since no respondeat superior could try intentional trespass - but no trespass because the damages were not immediate(not a direct trespass) what about substantial certainty --> So p went for strict liability Res ipsa (would not happen without d, negligence) v. strict liability - not concerned with negligence

Limitations of strict liability (not necessarily defenses, like privileges. ) Use of Firearms CASE: Miller v. Civil Constructors, Inc. (Ill COA, 1995) PG 699 p was injured by a stray bullet that caused him to fall out of his truck. The bullet originated from d's firearm practice in a gravel pit. P claimed activity was ultra hazardous and d should be held strictly liable
64

D cannot be held liable for injury to p due to d's ultra hazardous activities since d's activities were not ultra hazardous Restatement sec 520 Activity is Abnormally Dangerous/Ultra-hazardous if: 1-there exists a high degree of risk of some harm 2-if the likelihood of harm is great 3-if it is not possible to eliminate the risk by the exercise of reasonable care 4-if activity is not a matter of common usage 5-if the activity is inappropriate for the area of which it is carried on 6-if the extent to which its value to society is outweighed by it's danger Court finds that d's activity did not satisfy any of these factors: -difference in requiring a d to exercise reasonable care with a potentially dangerous activity than insuring absolute safety of others when engaged in ultra hazardous activity Also activity has use because it involved law enforcement officials who were practicing Ex: Hot air balloon in NYC injures a vegetable garden. P won b/c no matter how careful the D is, an accident will occur as a result of the activity. Incentives and Economic Analysis CASE: Indiana Harbor v. American Cyanamid (USCOA 7th, 1990) Pg 702 P suing d for recovery of expenses ($981,022.75) made to decontamination section of their rr line due to toxic leak of chemicals from one of the d's rr tanks. Tank was passing through a metropolitan area 1 count - d maintained tank car negligently 2 count - transporting such toxic materials was abnormally dangerous - strict liability should apply Distinction between the manufacturer and the actual shipment of material and then the distinction between the manufacturer's role in shipment The material itself cannot be dangerous it is the handling of the material that can be seen as dangerous- If reasonable care can be exercised to reduce the harm of an activity then there is no claim to be based on strict liability - Activity has to be unavoidably dangerous P should have sued on grounds of negligence and then provided proof of use of less than reasonable care, or sued the actual shipping company - the damages must be within reasonable limits Strict liability often comes up and succeeds in cases involving use of property Guille - hot air balloon applied the six restatement factors - Uncommon usage Strict liability forces the actor to find a cost effective way to do things safely - Posner - efficiency for the business - risk-utility analysis Limitations on Strict Liability -Scope of risk - damage that results from activity (if p activity is abnormally sensitive then d's liability is limited -third party intervenes - if unforeseeable then a superseding cause - relieves d of liability even when the damage is within the foreseeable risks -Unforeseeable Act of God -p's contrib neg not a defense, but unreasonable or sometimes reasonable assumption of risk is Also some juris reduce d's liability if comp neg can be shown Scope of the Risk - Proximate Cause Limitation - Remoteness of Risk of Harm -p doing something unusual also - so apply negligence standard CASE: Foster v. Preston Mills (SC of Wash, 1954) Pg 710
65

p ran a mink mill approx 2miles from where d was blasting. p's mink frightened by blasting done by d, mink killed her kittens. P had alerted d that the blasting was frightening his mink D not held strictly liable for the injury to p's minks when d was warned that his blasting operation was upsetting the mink proximate cause? - harm that is caused within reasonable limits and injury must result from that which makes the activity ultra-hazardous, but with strict liability is more concerned with policy Does causing a mink to be upset and kill her children make the blasting ultrahazardous? Actually risk of harm is damage to persons or property, no one else in the neighborhood was neg affected by vibrations or noise of blasting Blasting - strict liability should apply (Spano v. Perini). BUT does the risk of harm that subjects activity to strict liability carry fwd to the injury complained of. How does it connect to proximate cause - there is some negligence resonance in these cases. Think of strict liability as different language of negligence concepts. Similar to analysis of proximate cause - wagon mound direct causation but no liability. Keeping the mink itself is an extraordinary thing. Something about the Ps behavior makes it properly decided. Is keeping of mink farm subject to strict liability? Would have to see if the animals are vicious or that it is an extraordinary use of the land to have minks. Public policy - line is drawn d cannot be held limitlessly responsible for every consequence that result from the very things that cause the activity to be classified as ultrahazardous - Scope of the risk Act of God exception - compare to Ryland Fletcher (vis major, or the act of God) CASE: Golden v. Amory (SC of Mass, 1952) Pg 712 d's owed a hydroelectric plant with a dike, during a hurricane the river overflowed causing damage to ps property. d's plant was not permitted or approved by county, ps allege negligence also D not held strictly liable for damage to ps property when the activity in d's control, maintenance of a dike, became ultra-hazardous due to a hurricane The hurricane and subsequent flooding was the main ultra-hazard and was an act of god not in d's control Assumption of the Risk v Contrib Negligence Limitations CASE: Sandy v. Bushey (SC of Maine, 1925) Pg 714 P kept his colt and mare in d's pasture to graze with d's 3yr old colt. P was kicked by d's colt and sustained injuries D can be held liable for injury to p by d's horse when d's horse was on his premises and d had scienter of horse's vicious nature If the d knew his horse to have a vicious nature he should have exercised reasonable care - p would have to prove the scienter, the keeping and the vicious nature of the animal in order to prove strict liability. P did supply evidence of these things, strict liability does apply and no error was made in finding for the p Was p contrib neg? not applicable since strict liability is not negligence doctrine and therefore contrib neg cannot be used to counteract - but unreasonable assumption of the risk is a defense D would have to show that p was more than neg by interacting with animal to relive himself of liability as insurer Did the p unnecessarily and voluntarily interact with horse he knew was vicious, thereby bringing the injury upon himself? No, p had moved away from colt and colt still attacked him silently and swiftly (assumption of risk?) Some jurisdictions use comparative neg standard
66

7. PRODUCT LIABILITY Product liability - liability of a seller of chattel, which because of defects causes injury to purchaser, user or bystander. P purchase item directly from d and no contractual relationship Three theories: 1-Negligence - d failed to use reasonable care in manufacturing, designing, labeling etc product -can be liable to a remote purchaser or user or bystander (privity is not required, only foreseeability) 2-Strict Liability- seller of a product liable without fault for personal injuries if product sold is: 1-defective, or2-unreasonably dangerous. Non-manufacturer - retailers or anyone in distribution chain also included. Unavoidable unsafe products - drug that causes side effects. Warnings - failure to warn of some component that is non-obvious, warning of correct usage to avoid harm from incorrect usage. 3-Warranty: a.express - expressly warrant that goods have certain qualities - if not shown buyer can sue for breach of warranty (false claims). b.implied- mere fact that seller offered good for sale implies quality: Merchantibility - fit for the ordinary purposes for which such goods are used, Fitness for particular purpose - seller knows purchaser wants product for a particular purpose and buyer relies on the seller's recommendation of suitable product. 1-Negligence Privity - p must show that he contracted directly with d - Now not required to show privity due to introduction of middleman Rejection of Inherently Dangerous Standard - any product negligently manufactured - the manufacturer can be held liable for any injuries proximately caused by his or her negligence CASE: MacPherson v. Buick Motor (NJ)Pg 409 Cardozo D was car manufacturer, sold car to retail dealer who resold to p. Car collapsed and p was thrown out and injured. One wheel of the car was made with defective wood. Wheel was not made by d, d had bought it from another manufacturer. Evidence presented that d could have discovered defect with a reasonable inspection and there was none. No evidence that d knew of defect, not fraud but negligence D owes duty of care to p for injuries caused by defect in car sold by d when the product was inherently dangerous and was not inspected before sale. Inherent danger of car gives warning enough but the addition of a defect in a product that was not inspected before sale does show a want of reasonable care. Any harm from not inspecting can and should be reasonably foreseeable. Manufacturer was responsible for the finished product If possible to establish Negligence often basis used for product liability - jury access Retailer may also be sued for third party failure to inspect if there is a duty to inspect, or reason to know of danger 2-Warranty - elements of contract and tort (1) Express Warranty - Manufacturer/retailer explicitly saying s/t about the nature of the product upon which the consumer relies will be held liable even w/ no privity. (1) Liability in contract - regular misrepresentation would be tort. (2) Baxter v Ford Motor Co-D represented to the P that windshield was shatterproof. Injury to eye. Tort remedy even though cant prove negligence! (3) Caveat Emptor (let the buyer beware) no longer applies: (a) Buyer relies on manufacturer
67

(b) And mass advertising and marketing CASE: Baxter v. Ford (SC of Wash, 1932) Pg722 P bought a ford town car from a ford dealer who purchased it from the d ford motors. Both dealer and ford made representations to him that windshield glass was shatter proof. While driving car a pebble struck car windshield and glass shattered, flying into p's left eye and causing loss of sight ISSUE: did tr ct err in not allowing evidence of materials which could have been shown to be express warranty of shatterproof windshield when there was no privity of contract? HOLDING: yes REASONING: printed materials listed glass of ford car as triplex shatter-proof glass windshield. Such material should have been admitted into evidence With that the cause of action can be for breach of an express warranty - false advertisement to a passive p - Even with no privity of contract? - yes, policy reasoning Why couldn't they rely on macpherson - couldn't prove negligence - the windshield was up to the specifications (2) Implied Warranties Warranty of merchantability UCC 2-314 (1) - a warranty that goods shall be merchantable is implied in a contract for their sale if the seller is a merchant with respect to the goods 1-Merchantable - fit for the ordinary purposes for which such goods are used 2-Packaging - to be merchantable the goods must also be adequately contained packaged labeled and conform to the promises or affirmations of fact that are made on the label Merchant must be a businessperson and must regularly sell the goods in question Unclear whether there is implied warranty for used goods (a) Privity not needed: concern for consumer safety trumps K formation. (2) Henningsen v Bloomfield Motors Co- Still recover even though disclaimer (i) No negligence and no warranty, but liable b/c focus on the object and the object was bad. (ii) This case continues the trend in its emphasis in the passivity of the P. CASE: Henningsen v. Bloomfied (SC of NJ, 1960) Pg 726 p's wife injured while driving p's car due to failure of the steering mechanism. P purchased the car from d, bloomfield motors who purchased it from chrysler corp. p can recover from d for breach of implied warranty when she did not contract with the d for the goods directly. compared to implied warranty for food cases - implied that the goods sold were of reasonable quality - the warranty runs with the goods - so anyone who consumes defective food products can sue The unwholesome beverage may bring illness to one person; the defective car with its great potentiality for harm to the driver, occupants and others, demands even less adherence to the narrow barrier of privity Disclaimer is ineffective - unequal bargaining power made it an adhesion contract - ruled invalid (compare to Seigneur) P was not in privity because 1-chrysler sold the car through a dealer (vertical) and 2-her husband was the actual purchaser (horizontal) But she could still sue because she was a person who in reasonable contemplation of the parties to the warranty might be expected to become a user of the automobile (foreseeable p) 3-Strict Liability in Tort Due to the restriction of warranty liability by privity (or abandonment of privity requirement) many
68

courts have abandoned implied warranty for a strict tort liability i) PRIMA FACIE CASE: (1) Defect-must have been in a defective condition unreasonably dangerous to the user or his property (2) Control-when it left the Ds control was defective-needs to be exclusive control (3) Changes-product cannot be altered (4) Business-not a casual seller ii) Identification of the defect is enough to make the case go forward and it doesnt matter if the manufacturer was acting reasonably. iii) When a manufacturer puts a defective product on the market knowing that it will not be inspected for defects, it will be strictly liable for any injury caused by the defect in the product where it is used in the way it is intended to be used. ***CASE: Greenman v. Yuba Power Prod (SC of Cal, 1963) Traynor Pg 732 p bought a Shopsmith power tool from retailer, who bought it from manufacturer. After using the tool several times without incident while working on a piece of wood the wood flew out of the machine and hit him on the forehead. 10.5 mos later he brought suit against the retailer and the manufacturer for breaches of warranty and negligence P can recover for injuries from d's breach of express warranty and negligence when p did not give the d's notice of breach of warranty? failure to give notice of breach does not bar the p's action since manufacturer was in strict tort liability -Manufacturer placed an article on the market knowing that it was not inspected for defects - and the product then turned out to have defects that caused injury to the p -Notice of breach requirements (Sec 1769 of the Civil Code) are only suitable for commercial transactions and not for ordinary consumers who do not have a contract of sale -Purpose of such liability is to insure the costs of injuries resulting from defective products are borne by the manufacturer rather than by injured consumers who are powerless to protect themselves Sufficient for p to prove that he was injured while using the product for what it was intended to be used and due to a defect in the product he was not aware of pg 734-745 Restatement sec 402A - Special liability of Seller of Product for Physical harm to User or Consumer (1) One who sells any Product in a defective condition unreasonably dangerous to the user or consumer or to his property is subject to liability for physical harm thereby caused to the ultimate user or consumer, or to his property, if (a) the seller is engaged in the business of selling such a product, and (b) it is expected to and does reach the user or consumer without substantial change in the condition in which it is sold. (2)The rule stated in Subsection (1) applies although (a) the seller has exercised all possible care in the preparation and sale of his product, and (b) the user or consumer has not bought the product from or entered into any contractual relation with the seller. Consumer Expectations Test - defective condition unreasonably dangerous - dangerous to an extent beyond that which would be reasonably contemplated by the ordinary consumer who purchases or with the ordinary knowledge common to the community as to its characteristics. -if a reasonable consumer knowing the characteristics of the product would nonetheless use it the product is not in a defective condition unreasonably dangerous. Contrast California rule with restatement 402a
69

-p still needs to show some kind of defect in the product Cal doesn't talk about defect being unreasonably dangerous - with strict liability products dangerousness would be unavoidable - no knowledge on the part of the d of the defect is considered CA doesn't talk about the exercise of all possible care - more p friendly - no need to show evidence of d's lack of reasonable care P Favorable<------------------------------------------------------------------------------------> D Favorable NJ CA 402A MI (think of car factories) 3 Types of Product Defects: Third Restatement: Topic 1. Product Defectiveness Sec1. liability of commercial seller or distributor for harm caused by defective products (a)one engaged in the business of selling or otherwise distributing products who sells or distributes a defective product is subject to liability for harm to persons or property caused by the defect. Sec2. categories of product defect A product is defective when, at the time of sale or distribution, it contains a manufacturing defect, is defective in design, or is defective because of inadequate instructions or warnings. A product: (a) contains a manufacturing defect when the product departs from its intended design even though all possible care was exercised in the preparation and marketing of the product; (b)is defective in design when the foreseeable risks of harm posed by the product could have been reduced or avoided by the adoptions of a reasonable alternative design by the seller or other distributor, or a predecessor in the commercial chain of distribution, and the omission of the alternative design renders the product not reasonably safe; (c)is defective because of inadequate instructions or warnings when the foreseeable risks of harm posed by the product could have been reduced or avoided by the provision of reasonable instructions or warnings by the seller or other distributor, or a predecessor in the commercial chain of distribution, and the omission of the instructions or warnings renders the product not reasonably safe. Takes away: -need for p to show unreasonably dangerous for manufacturing defect Adds on the: -negligence basis and risk-utility analysis weighing the risk of harm from the unavoidable inherent danger of the product against the utility of the product (whether the product is reasonably safe) for design and warning defects -P can introduce a reasonable alternative design (A) Manufacturing Defect Strict products liability - p favorable CASE: Rix v. General Motors Corp. (Mon, 1986) p's pickup truck was hit from behind by a truck with a water tank manufactured by d. P sued on theory of strict liability for manufacturing and design defects Brake failure - manufacturing defect in the brake tube Brake system was defectively design - should have had a dual braking system instead of a single GMC agreed to brake tube defect but claimed tampering after it left them - not their responsibility And single system not defective - accident may have occurred anyway Court gave jury two instructions: 1-Strict liability doctrine of Restatement 402a 2-3elements needed to prove strict liability:
70

1-d manufactured and sold a product which at the time of sale was in defective condition unreasonable to user 2-product was expected to and did reach the ultimate consumer without substantial change 3-defective condition proximately caused p's injury D can be held strictly liable for p's injuries when the product which led to injuries had manufacturing and design defects. The essential question is whether product was flawed or defective due to improper construction by manufacturer Manufacturing defects are: defects in a particular item different from the other same items manufactured because something went wrong during the manufacturing process (a lemon statistically inevitable) Does not conform in some specific way to the intended design nor does it conform to the great majority of products manufactured with that design. (mishap in the manufacturing process itself, improper manufacturing, or use of defective material) Compare to defective spokes in MacPherson and failed steering gear in Henningsen (B) Design Defect Elements of negligence standard - d favorable All of the similar products manufactured by the manufacturer are the same and they all bear a feature whose design itself is defective/unreasonably dangerous - usually examined under a pure negligence standard - a product's utility will be weighed by the risk of harm it may cause CASE: Prentis v. Yale Mfg. Co. Pg 743 (MI, 1984) p was working with a stand-up forklift manufactured by d. P fell off of forklift while operating it and injured his hip. p sues claiming the forklift defectively designed due to a lack of a seat for the operator A p can recover for injuries due to a manufacturer's product being defectively designed under a negligence standard (Model Uniform Product Liability (UPLA)) Tr ct judge instructed jury under a negligence standard rather than a strict liability standard Question was whether design of forklift was unreasonably dangerous since it lacked a seat Such defective design like lack of a seat or platform for the operator could have been foreseen to cause injuries (unreasonable risk of harm): 1-negligence standard rewards the careful manufacturer and penalizes the careless one - encourages design of safer products 2-finding a defective design will get rid of an entire line of defective products - may lead to bankruptcy of manufacturer - higher threshold of liability needs to be used 3-use of fault brings an intrinsic fairness - careful manufacturer will have to bear the costs of a negligent product seller (proximate cause may cut off their liability) Risk-utility analysis - key issue is the reasonableness of the manufacturer's conduct in placing the product on the market: 1-relative need for the product 2-likelihood product will cause harm and the severity of such probable harm 3-availability of an affordable safer alternative design Alternate tests: combined negligence and strict liability test for design defects: Design defect if Either: 1-design's dangers outweigh its utility (risk-utility analysis - negligence) or
71

2-design does not perform as safely as a reasonable consumer would expect when used for an intended or reasonably foreseen purpose (strict liability - consumer expectations test) Even with the exercise of reasonable care could not prevent the harm - such as a product that is completely flawed in design and not in part (pg 700) - if reasonable care could have been used then we should look at negligence Cal would have probably came out differently - would have tried to use the UPLA This contains a shift to manufacturer's conduct and Negligence is d favorable - harder for p to prove all elements Design Defect - Risk-utility analysis and RAD p-friendly - even if no RAD shown risk-utility says discontinue product CASE: O'Brien v. Muskin Corp (NJ) P dove into pool manufactured by d and was injured when his hands slid on the vinyl lining of the pool causing him to hit his head on bottom of pool Pool was20x24' and water depth was 3.5', logo on pool saying "Do Not Dive" P claims defective design of pool due to slipperiness of vinyl lining and lack of adequate warnings P can recover for injuries sustained in pool manufactured by d under strict liability when pool was defectively designed even if it meets the state-of-the-art standard tr ct instructed based on theory of strict liability concerning the warning only - took away consideration of vinyl lining as a manufacturing or design defect For strict liability of vinyl lining defect P has to prove product was defective And defect caused injury to reasonably foreseeable user Pg 751 Design defects - standard based on the policy - risk-utility analysis: 1-utility to user and public 2-safety aspects 3-reasonable alternative design 4-feasibility of safety improvements 5-users ability to avoid danger by using reasonable care 6-users awareness of products dangers 7-manufacturers ability to raise price or get insurance 8-state-of-the-art at the time the product was manufacturer (C) Warning Defects - Concealed danger with inadequate warning Duty to warn about defects - presence or absence of a warning as to the possible dangers of a product may have a great bearing on whether the product is "defective" or "unreasonably dangerous" as may the quality of the directions for use given by the manufacturer Warning is only an extra obligation placed on the manufacturer - does not replace design or manufacturing defects Duty to Warn of Unknown and Unknowable Dangers CASE: Anderson v. Owens (Cal. 1991) p developed asbestosis from exposure to asbestos while working at a shipyard around area where insulation products were being removed and installed state of the art evidence can be used to defeat liability when the strict liability failure to warn standard is used
72

Knowledge or knowability is a component for strict liability for failure to warn Strict liability functioned to relieve the burden of proof of those injured Negligence in strict liability looks at the conduct of the manufacturer instead of the condition f the product Warning defects may be more rooted in negligence - warning HAS to include knowledge or knowability because not possible to warn of something unknown As opposed to negligent failure to warn - manufacturer fell below standard of care of a reasonably prudent manufacturer and what they would have known and warned about Use failure to warn in strict liability - spread cost of risks and injury to party more able to bear them - but not to make them the insurer of the safety of their products? BUT warning not a good substitute for a reasonably safe design And most jurisdictions have no duty to warn of obvious dangers or known risks - such knowledge can be assessed by the jury or factfinder as well as the adequacy of any warnings Proof of Causation CASE: Friedman v. General Motors Corp. (SC of OH, 1975) pg 765 p turned ignition key of a 17 yr car will in drive gear - car started and crashed. P suing gm - the manufacturer there was sufficient evidence to prove vehicle defective P only has to show: 1-car was manufactured by d and was defective 2-defect existed at the time car left the factory 3-defect was the proximate cause of the accident and injuries Circumstantial evidence is enough - more likely than not the cause The parts were the same parts present when car left factory Might have been the first time car was started in drive gear Manufacturing cases do not use res ipsa but using an analogy to res ipsa (in the absence of a defect v in the absence of negligence?) Dissent Proof of defect is often difficult Usually proof of defect established with circumstantial evidence and testimony - product itself is sometimes destroyed in the accident But this is not enough - also need proof of products condition before accident as well as expert testimony Only in cases of res ipsa where the accident could not have happened without a defect does circumstantial evidence work Otherwise could have been proven with: 1-expert 2-testimony of damaging event 3-damaging event and expert testimony 4-evidence and probable cause -as well as negating of other probable causes 5-physical evidence of product after accident Escola v. Coca Cola - exploding coca cola bottle - court used res ipsa loquitur / Traynor felt more so strict liability claim 8. PRODUCT LIABILITY DEFENSES
73

(A) Plaintiff's Conduct i) Assumption of Risk -Arguments have been made that it is less of a defense and more of a problem with the claim that the P is bringing that the P relieves the D of his duty to protect him, so no cause of action! (1) Let the jury weigh the fault of the D in strict liability cases. (a) COMPARATIVE NEGLIGENCE IS A DEFENSE . (Prof says that can argue that it shouldnt be) ii) Instructions/ Information iii) Misuse of the product (1) Ford Motor Co v Matthews (a) Restatement comment f: A product is not defective when the product is misused! (2) Where the misuse is foreseeable, then the D can still be held liable. (B) Exception to strict liability : Prescription drugs-design defects-because of their incredible utility. i) 402a-k-unavoidably unsafe products often form an exception (2) Where the misuse is foreseeable, then the D can still be held liable. Negligent Failure to discover the defect Intentional assumption of the risk from the defect CASE: Daly v. General Motors Corp. (SC of Cal, 1978) pg 769 defective door latch, during accident p was thrown from his car, but wasn't wearing a seatbelt and was intoxicated judgment reversed A manufacturer can be found strictly liable when the p was comparatively negligent Comparative negligence may stem from contrib neg against a negligence claim , originally not a defense, but should be able to be used as a defense for strict liability in part as well Li - loss should be assessed equitably in proportion to fault For strict liability assumption of risk is a complete bar to recovery for p But if merged into comparative negligence as negligent assumption of risk - then only partly reduces recovery in a few jurisdictions Misuse of a Product: 1-may reduce p's recovery 2-may indicate no defect 3-may show defect not the proximate cause of injury Reasonably Foreseeble Misuse - if could have been design3ed without undue cost or significant changes the product will be looked at as defective CASE: Ford Motor Co. v. Matthews (SC of MS, 1974) P 774 decedent was dragged underneath tractor - he was standing beside tractor when he started it and tractor was in gear - tractor had a safety switch that p's estate claimed defective Manufacturer not held liable for product defect when d misused the product but the use of product was reasonably foreseeabble adds limitation on ford's liability if product is misused - looking at restatement 402a Most times where this was effective was when the court did not find a defect or defect was not the proximate cause But the failure of the safety switch can be found to be a cause of the p's death - failure to make sure tractor was in neutral was just omission of a customary precaution - reason why there is a safety key! If such use was not reasonably foreseeable then manufacturer not liable (compare to Muskin)
74

Here danger was foreseen Unavoidably unsafe products will not subject manufacturer to strict liability - strict liability is for products that can't be made any safer 9. DEFAMATION 1) Interest protected: Ps truthful and accurate reputation in the relevant community. 2) Defamation is a communication that damages the Ps reputation by diminishing respect, good will, confidence, esteem he is held, or to excite adverse or unpleasant feelings about him. 3) PRIMA FACIE CASE: a)Ps Burden of Proof at Common Law: (Strict liability because no fault or intent) 1- Defamatory language-utterance tending to lower Ps reputation 2- Publication-communication to 3rd persons (at least 1 person besides P) 3- Colloquium-the remark is of and concerning the P b) Common law does not require a showing of but Constitution may: 4-Damages (except by non-slander per se and libel per quod) 5-Falsity 1-Defamatory Language: CASE: Belli v. Orlando Daily Newspapers (USCOA 5th) d printed a newspaper article that stated that p and his wife (tort lawyer melvin belli) while on all expense paid appearance racked up hundreds of dollars on clothing whether trial court cannot dismiss a libel suit based on interpretation that statement had a nondefamatory meaning when it can be interpreted that there could be a defamatory meaning as well if there could be at least one possible reasonable meaning that is defamatory then should go to the jury for them to decided if at least one person would take that defamatory interpretation Libel per se is any written publication that exposes a person to distrust hatred contempt ridicule or obloquy, Through false and unprivileged publication by letter or otherwise which can injure a person in his office occupation business or employment Judge decides of words are injurious and Jury determines if defamatory-ness conveyed - through inducement or innuendo Not required that reader believe it to be true as long as they would interpret the statement as trying to convey a defamatory meaning Strict liability case - no fault shown -Utterance has to be pleaded in specificity - this is the thing held strictly liable n.1 Lowering the reputation of the plaintiff tending to hold the plaintiff up to obliquy (hatred, contempt or ridicule or to cause him to be shunned or avoided) CASE: Grant v. Reader's Digest Ass'n Judge Learned Hand d a magazine printed story where lawyer grant (p) stating that lawyer was a legislative rep for the mass commie party A statement can be interpreted as defamatory even if only a small minority of people would hold the statements allegations against the p Certain allegations may only diminish a person's rep in certain segments of the community but a statement is defamatory as long as a significant and respectable minority of persons draw an adverse opinion of the p from it, even if most people would not
75

Mawe v. Piggot - respectable group Pg 835 n.2 Peck v. Tribune - Issue embedded in time place and culture, although the statement will stand by itself, Must be an element of discredit or disgrace even in the eyes of a particular segment of a community Pleading defamation If defamation on its face p must plead: 1-words themselves 2-communication to third party If not on its face: Pg 836 n.6 P must show context that renders words/conduct defamatory 3-Also must show words were spoken of and concerning the p - colloquium, an allegation of innuendo meaning conveyed by the words, and damages Pg 837 he burned down his own barn - may have to show inducement or innuendo to show the defamatory nature of this statement, otherwise would seem too benign to get to a jury trial Establishment of Colloquium & Defense of substantial truth CASE: Kilian v. Doubleday (SCPenn, 1951) course in english taught to disabled wwii vets, wolfe published book of essays written by students through doubleday -d One of the stories written by a joseph o'connell wrote about an abusive sergeant while at an army hospital which referred to colonel kilian - p Wolfe put addendum stating kilian was convicted of permitting cruelty to soldiers But kilian was only convicted of neglect but not of any actual wrongdoing on his part D pleaded affirmative defense of truth but on stand d claimed he was never at lichfield - so couldn't use because article was false and he also could not show evidence of the incidents described having actually took place - only need to show claims where substantially true but could not D cannot use the defense of substantial truth of statements when d's article was false Not necessary for statement to be literally true in all respects - it must merely be substantially true Proof of different offense - but if d has made a charge of wrongdoing against p then he may not establish truth by proving a materially different charge - d accused p of knowingly encouraging the mistreatment of american soldiers, the fact that d was convicted of mere neglectful permitting of such mistreatment was not enough to establish the defense of substantial truth Burden of proof - p has to show statement false - then no truth defense for d to use Two issues in Kilian: 1-colloquium - colonel was identifiable (contrast with bindrim where colloquium is a much bigger problem) 2-truth defense (affirmative defense of justification) Pg 839 - not easy to prove truth and burden is placed on d/ what if p had burden of proof of falsity - in seditious libel - truth was not a defense - and actual true statement may have ended up hurting d more than a false statement pg 840 Pg 840 n.1-2 Colloquium Defamatory statement about a group/class - cause of action depends on the size of the group or class referred to CASE: Neiman Marcus v. Lait ( SDNY, 1952) d's lait write a book usa confidential in which they say that most of the salesmen at neiman marcus are fairies - there were 25 salesmen, and some of the models were call girls, and salesgirls also call girls 1-9/9 models, 2-15/25 salesmen, 3-30/382 saleswomen
76

Individuals can bring a defamatory suit when the defamatory statement refers to some or less than all of a group or class 1-Where the group/class libeled is large none can sue - individual within group has a hard time to prove statement refers to that individual 2-where the group or class is small and each and every member is referred to then any individual member can sue Saleswomen group too large for statement to refer to any of the individuals individually - dismissed Models and salesmen suits (and corporate suit) can go ahead If reference to only part of a class then p may not be able to recover unless p shows that a substantial portion of the group was included - use of the word 'most' May be defamatory to each individual within the class since the size of the class was small In some jurisdictions - Size alone may not be determinative but it can be used as a factor in looking at the intensity of suspicion casted upon an individual Reference need not be by name 2-Colloquium P defamed as a fictional character - colloquium need not be by name if can reasonably be understood to refer to p CASE: Bindrim v. Mitchell p conducted nude marathons with his therapy group. D was in the group and afterwards wrote a novel called touching where the principal character mirrored the p. D had signed a contract with p not to take photos write articles or in any manner disclose who has attended workshop or what has transpired. P claimed defamation And sued the publisher and author. D says not identifiable as him d statement was defamatory to p when d did not mention p by name but d could reasonably be identified by people who knew him The differences where that p and character named herford have different physical appearances and professions. Even if novel the character was identifiable as the p despite slight differences Test is whether a reasonable person reading the book would understand that the fictional character described is the p - p doesn't have to be named - Jury question Also third party can be just one other person doesn't have to be a group who would recognize d Concurring opinion - 1st amendment right to comment does not include the right to commit libel -Why not just based on breach of contract? Probably p did both but we are looking at defamation claim To label a work fiction does not excuse author from liability for defamation What if book had a disclaimer of references to person living being coincidental? Doesn't excuse liability for defamation Kalven Paradox - Prof Harry Kalven - in dissent - there is revealed here a new technique by which State that the statement actually refers to the p but that the statement is false - easy to prove about a statement that doesn't refer to the p in the first place Pg 848 P helped his colloquium by saying that he is a public figure and brought plausibility to his argument that character could only be referring to him - but also brings upon himself a constitutional burden Libel and Slander Now libel - sight- written or printed words, however they are conveyed and slander - hearing (also hand gestures) and everything else not in libel category CASE: Shor v. Billingsley Pg 852
77

adlibbed remark "I wish I had as much money as he owes" (compare to pg 834 n.3 remark), on radio broadcast p suing for multiple counts including libel and slander defamation statement on a radio show is a claim of libel when the statement was not scripted libel because of the durability of the medium (radio) wider reach Radio broadcasts read from a script - libelous Not read from script it is then slander but damages should be assessed still under a libel standard because of its wide reach (publication power) Now primarily regulated by statute - most view broadcasts as slander whether scripted or not Why are libel and slander treated differently? Slander as oral not held to be serious (gossipy), written word may have more seriousness (gravitas) Permanence of libel (durability) -Slander may bring in problems of proof/evidence / but still can find witnesses -not that spoken statement can't still be hurtful - but looking more at the lowering of a reputation, spoken words seem to be less permanent/ disappear rapidly Defamation has to be pleaded and proved with great precision Special damages for slander that is not slander per se CASE: Terwillinger v. Wands Pg 857 action for slander - p proves that d made defamatory comments (rumors) about p possibly having a relationship with a mrs fuller whose husband was in jail and that he was trying to keep the husband in jail - p suffered and was unable to attend to business P cannot claim special damages for slander when he did not suffer pecuniary loss Special damages must be the natural, immediate, and legal consequence of the words Court states the injuries were only to reputation and with words that are not slander per se - do not appear on their face to be defamatory - there has to be some pecuniary loss(loss of money, or things that have monetary value) to prove injury to p's reputation Not slander per se - referring to a man being sexually immoral - reflects the time of the case Needing to prove that you were damaged actually makes p's case harder Terwilliger only showed emotional distress damages - do not show proof of harm to reputation! Doesn't have to be economic in the pure sense - could be shunning/obloquy Injury to p's reputation: Harder to state defamation for spoken statements (slander) so add to p's burden of proof 1- all libel is Libel per se injury is presumed for all libels - no need to show special damages through pecuniary loss (defamatory statements incorporated in a writing is presumed) In some jurisdictions If libel not on its face then libel per quod - p must allege and prove extrinsic facts in order to show defamatory character and some even require a showing of special damages (weisberg thinks this is a mistake and won't be testing on this) (fault, falsity, and damages were assumed (truth was a d's defense)) 2-slander - spoken statement - injury is presumed for slanders per se otherwise not presumed -if not - p must show injury to reputation (e.g. lost job or customers - pecuniary loss Terwilliger v. Wands) (fault, falsity and damages not assumed)
78

-slander per se - goes to the effect of the defamatory statement : 1-any defamatory statement that is negatively directly related to p's business or profession 2-any defamatory statement attributing criminal activity to p (moral turpitude) 3-loathsome disease 4-serious sexual misconduct - normally a charge imputing unchastity to a woman 3-Publication Communication of defamatory statement to anyone besides the person being defamed - even one other person Can't just be said to the p - doesn't lower p's reputation in anyone else's eyes - essence of this tort is injury to reputation -Republication - may be held liable for repetition that may have been reasonably foreseen Although party repeating statement can also be held liable CASE: Economopoulos v. AG Pollard 1-slander - falsely and maliciously accusing p of theft D said it in english and greek clerk translated it to p/ except for clerk no third party heard the statement there is not publication of a defamatory statement when the statement is translated no publication in english, not spoken in front of any one else in a language that they could understand, Statement has to be overheard by others - Translator is an extension of the speaker Pg 859 n.4 Typically an agent of a speaker considered the same as the speaker or the person spoken to: -Also same for stenographers or transcriptionists -Internal communications and qualified privilege Pg 859 n.6 Communication can be intentional or negligent - d has to be aware or foresee third party hearing -concealed third party - not considered publication - absence of intent or substantial certainty to publish Pg 870 Hulten v. Jones "Whist! There is Artemus Jones with a woman who is not his wife, who must be you know - the other thing!" -Actually made up the name but there actually was an artemus jones - he was permitted to sue Secondary Publishers - not liable if they had no knowledge of libelous matter or has no reason to be put on guard But still need a general understanding of defamatory-ness for colloquium but only 1 person for publication Identity Theft - Invasion of Privacy CASE: Carafano v. Metrosplash.com, inc. (9th, 2003) matchmaking service online stautorily immune from invasion of privacy so long as the information was provided by a third party pg 861 Statute of Limitations for Defamation Runs from date of Original Publication (Single Publication Rule)Pg 867-879 CASE: Ogden v. Assoc of the United States Army(DC, 1959) book originally published in 1955, suit filed in 1959, sol in dc for defamation is 1 yr A new suit for libel does not arise for each new sale or delivery of material containing the libelous statement(s)
79

single publication rule - sol for defamation runs from the date of original publication -and gives rise to only one cause of action for libel Overruled duke of brunswick v. harmer - each sale or delivery of offending material gives rise to a new cause of action - bought copy 17 years after original publication -publication in different sources are viewed as separate publications, and republication/new editions counts as a new publication -internet - can be constantly modified - not under publication rule with republication constraints First Amendment Concerns (A)Public Plaintiff - Malice to be shown in cases of defamation against a public figure CASE: New York Times v. Sullivan RULE/ANALYSIS: The Supreme Court held that petitioner was protected by the First and Fourteenth Amendments. As such, a public official [respondent] was prohibited from recovering damages for a defamatory falsehood relating to his official conduct unless it could be proven that the statement was made with actual malice. (knowledge of falsity or reckless disregard for falsity) 2-in a matter of public concern the p cannot recover damages without showing actual malice on the part of the d SUMMARY: Respondent presented no evidence to show petitioner was aware of erroneous statements or was reckless in that regard, and therefore could not prove malice. In the absence of malice, respondent could not recover damages. Usually d had the burden of proof (justification) but now looks like p had that burden Gertz pg 898 Only step in to state matters when there is a constitutional reason - In this case looking at the first amendment To say that a statement is a falsehood mean that the person who uttered the statement to have knowledge or knowability - intent or negligence introduced For libel suits p's usually were satisfied with minimal award as long as they won - set the record straight/rebuilds their reputation to where it was before the defamatory statement lowered it Public Policy Concerns of Defamation Suits: Possible scotus is looking at ny times v. sullivan because of: -the huge award amount -if the defamatory statement was accurate - then no liability on the part of d -Issue of falsity of the statement/not the lowering of the reputation of the p -constitutional law Colloquium issue - public figures/public officials - public should have the right to make comments concerning them without worrying about defamation under the constitution General critique of government could be transformed in tort into a private infringement and stifle public discourse - biggest public concern at core of 1st amend is the right to criticize government Gets rid of the possibility of seditious libel suit Includes Public figures like celebrities - burden is hard for them to prove defamation - wrong move? What is constitutionally troubling about the burden of proof of truth being on the defendant? First amendment permits free speech - D has the right to justify their statement Ensure that debate on public issues will be uninhibited, robust and wide open First amend does not protect just true free speech, it protects all speech Chilling effect of the burden on d's - leads to self-censorship/ d can't always know if statement is true
80

Pg 876 n.2 maybe we are only concerned about diminishing the chilling effect and not putting too much burden on the d - Fair Comment on matters of public concern Times had a chance to defend by showing truth - common law also had privileges though where d had a privilege and did not have to show truth Coleman - minority view even false statements about a public issue is privileged if made in good faith In this tennis match after d states privilege p can defuse it by showing d made statement in bad faith (malice - 1-spite or intent to harm p (d's relationship to p) - not constitutionally upheld, 2-knowledge of falsity or reckless disregard for falsity(d's relationship to the utterance)) Any living person can be defamed, defamation of the dead may however defame a living person Corporation defamation? - no, neither can partnerships, not for personal senses, only if statement casts an aspersion on its honesty, credit, efficiency, or business/moral character But governmental institutions usually have no right to sue in libel - should be open to uninhibited public criticism (B)Private Plaintiff - Public Concern CASE: Gertz v. Robert Welch Inc. policeman Nuccio shot and killed Nelson and was convicted. Nelson's family brought civil suit with attorney Gertz. Respondent robert welch published a magazine that contained statements accusing gertz and others of conspiring to discredit law enforcement as part of a communist conspiracy. Accused nelson civil trial of bringing false testimony against nuccio and being the architect of the frame-up. Gertz claims not to have discussed nuccio with press nor to have taken part in crim proceedings. d's can claim a constitutional privilege for defamatory statements when the p was not a public figure or official if the statement was of a matter of public concern Under 1st amend - no false ideas/opinions - but false statements have no constitutional value/importance - still inevitable that there will be some during discussions - to punish people from making such statements would limit free speech by making people more cautious and restrictive To impose strict liability standard for defamation would be too burdensome - introduce an element of fault NY times defines constitutional protection for a public figure or official - in order to prove liability for defamation they have to prove that the defamatory statement was false and was made with the knowledge of its falsity /or with reckless disregard for the truth - heightened burden of proof for public person plaintiffs Public persons have more means of correcting/contradicting a defamatory statement than a private person (private persons are therefore more vulnerable to injury and their burden of proof should be lower - state interest in protecting them is greater) Assumption of risk theory - Public persons voluntarily accept the risk of greater public scrutiny and defamatory statements - rosenblum standard of including issues of public interest not accepted - left for individual states to decide/ public figures also have greater access to media to correct their reputation without litigation To be classified as a public figure - have to have clear evidence of general fame or notoriety in the community and pervasive involvement in the affairs of society - gertz not a public figure -private persons - no longer strict liability for defamation - private p now has to show at least some fault - negligence/ also damages no longer assumed - actual damages have to be shown
81

But to get special damages - punitive damages - had to show malice Pg 898 NY requires a showing of gross negligence Speech of Private Concern - Common Law Defamation Case: Dun & Bradstreet, Inc. v. Greenmoss Pg 902-907 Hs student working for greenmoss posted negative credit report of dun and bradstreet- credit report was incorrect Presumed and punitive damages awarded d's argue that the matter is of public concern and dun and bradstreet need to prove actual malice in order to get damages Court finds the matter is not one of public concern - only has to show common law requirements of defamation Dissent p should have to show actual malice to recover punitive damages Just as we saw in products liability a move that is cyclical - towards easing the p's burden but then moving back towards d - so in defamation same cyclical pattern of burden shifting in terms of first amendment concerns (F) Opinion Pg 915-922 Implied assertions of fact in an opinion piece CASE: Milkovich v. Lorain Journal Co. Diadun writing for lorain journal implied that p milkovich a wrestling coach lied under oath concerning a match Coa issued summ judg against p - opinion piece considered privileged from liability for defamation under state law and first amendment. scotus says no A defamatory statement made in an opinion piece is not protected from liability under 1st amend and state law. Scott court - coa used four factors to determine whether fact or opinion: 1-specific language used 2-verifiable statement 3-general context of statement 4-broad context in which statement appeared And found it to be an opinion piece Scotus rejected claim that statements of opinion can be completely exempt under the constitution from defamation liability, to be defamatory a statement must be provable as false - but that does not mean that there should be a wholesale defamation exemption for anything that may be labeled opinion - if a statement implies an assertion of objective fact, then there can be defamation liability for that implied assertion of fact if it is false.

82

S-ar putea să vă placă și